You are on page 1of 104

Page 159 of 525

Page 160 of 525


Page 161 of 525
Page 162 of 525
Page 163 of 525
Page 164 of 525
Page 165 of 525
Page 166 of 525
Page 167 of 525
Page 168 of 525
Page 169 of 525
Page 170 of 525
Page 171 of 525
Page 172 of 525
Page 173 of 525
Page 174 of 525
Page 175 of 525
Page 176 of 525
Page 177 of 525
Page 178 of 525
Page 179 of 525
Page 180 of 525
Page 181 of 525
Page 182 of 525
Page 183 of 525
Page 184 of 525
Chapter
15
Estimating
probabilities

Contents: A Probability by experiment


B Probabilities from tabled data
C Probabilities from two way tables
D Chance investigations

Page 185 of 525


100

100

100

100
50

50

50

50
0

0
25

75

95

25

75

95

25

75

95

25

75

95
5

IB MYP_3
cyan magenta yellow black

Y:\HAESE\IB_MYP3\IB_MYP3_15\315IB_MYP3_15.CDR Monday, 26 May 2008 3:12:00 PM PETER


316 ESTIMATING PROBABILITIES (Chapter 15)

OPENING PROBLEM
A triangular prism is made of plastic
with the dimensions shown.
2 cm
When it is thrown in the air it could 2 cm
come down and finish on: 2 cm
a triangular face a square face a rectangular face

Things to think about:


² Which side do you think the prism is most likely to finish on?
² How can we estimate the probability of the prism ending up on a triangular face?
² How can the estimate be improved?

PROBABILITIES
The probability of an event happening is a measure of the likelihood or chance of it
occurring.

Probabilities can be given as percentages from 0% to 100%, or as proper fractions or decimal


numbers between 0 and 1.

An impossible event which has 0% chance of happening is assigned a probability of 0.


A certain event which has 100% chance of happening is assigned a probability of 1.
All other events are assigned a probability between 0 and 1.

The number line below shows how we could interpret different probabilities:
not likely to happen likely to happen
0 0.5 1

impossible certain
very unlikely equal chance of happening very likely
to happen as not happening to happen

The assigning of probabilities is usually based on either:


² observing the results of an experiment (experimental probability), or
² using arguments of symmetry (theoretical probability).
In general, the probability of an event occurring is

number of ways the event can happen


P(event) =
total number of outcomes
Page 186 of 525
100

100

100

100
50

50

50

50
0

0
95
25

75

95

25

75

95

25

75

25

75

95
5

IB MYP_3
cyan magenta yellow black

Y:\HAESE\IB_MYP3\IB_MYP3_15\316IB_MYP3_15.CDR Monday, 26 May 2008 1:44:42 PM PETER


ESTIMATING PROBABILITIES (Chapter 15) 317

HISTORICAL NOTE
A dispute between gamblers in the 17th
Century led to the creation of a new
branch of mathematics called theoretical
probability.
Chevalier de Méré, a French aristocrat and a gambler,
wanted to know the answer to this question:
“Should I bet even money on the occurrence of at least
one ‘double six’ when rolling a pair of dice 24 times?”
De Méré’s experience of playing this game convinced
him that the answer was yes.
He asked two French mathematicians, Blaise Pascal and
Pierre de Fermat, to prove that his theory was true.
Not only did they correctly argue the result, but they became interested in solving other
questions of this kind.

A PROBABILITY BY EXPERIMENT
The probabilities for some events can only be found by experimentation.
Here are some words and phrases which we need to understand:
² The number of trials is the total number of times the experiment is repeated.
² The outcomes are the different results possible for one trial of the experiment.
² The frequency of a particular outcome is the number of times that this outcome is
observed.
² The relative frequency of an outcome is the frequency of that outcome divided by the
total number of trials.
The more times an experiment is repeated, the better the estimate of the probability of the
event.

EXPERIMENTAL PROBABILITY
Consider the Opening Problem again. A prism was made in this exact shape and was thrown
in the air 200 times.
The results were: Rectangular 77, Square 85 and Triangular 38
We say that:
² the number of trials was 200
² the possible outcomes were Rectangular, Square or Triangular.
² the frequencies were 77 for Rectangular, 85 for Square and 38 for Triangular
77 85 38
² the relative frequencies were 200 for Rectangular, 200 for Square and 200 for
Triangular.
Page 187 of 525
100

100

100

100
50

50

50

50
0

0
95
25

75

95

25

75

95

25

75

25

75

95
5

IB MYP_3
cyan magenta yellow black

Y:\HAESE\IB_MYP3\IB_MYP3_15\317IB_MYP3_15.CDR Monday, 26 May 2008 1:44:45 PM PETER


318 ESTIMATING PROBABILITIES (Chapter 15)

We use the relative frequencies, usually written as decimals, to estimate the probabilities.

The estimated experimental probability is the relative frequency of the outcome.

We write: P(Rectangular) ¼ 0:385, P(Square) ¼ 0:425 and P(Triangular) ¼ 0:190


When the experiment was performed again the results for 250 repetitions were: Rectangular
90, Square 112 and Triangular 48.
This time P(Rectangular) ¼ 0:360, P(Square) ¼ 0:448, P(Triangular) ¼ 0:192

DISCUSSION EXPERIMENTAL RESULTS


Discuss: ² why the two repetitions of the experiment of the Opening
Problem give different results and what this may be due to
² the ways these results may be combined.

Example 1 Self Tutor


Find the experimental probability of:
a tossing a head with one toss of a coin if it falls tails 36 times in 80 tosses
b rolling a pair of sixes with a pair of dice given that when they were rolled 300
times a double six occurred 9 times.

a P(a head) b P(a double six)


¼ relative frequency of a head ¼ relative frequency of a double six
9
¼ 36
80
¼ 300
¼ 0:45 ¼ 0:03

EXERCISE 15A
1 Jacinta has been given an indoor golf hole to practice her putting. Out of her first 91
attempts to hit the ball into the hole, she succeeds 23 times. Find the experimental
probability of Jacinta hitting the ball into the hole.
2 When John makes an error in his work he crushes the sheet of paper into a ball and
throws it at the waste paper basket. At the end of the day he has scored 16 hits into
the basket and 5 misses. Find the experimental probability that he scores a hit into the
basket.
3 In the first round of a competition Sasha recorded Find the best estimate
by adding the
77 hits out of 80 shots at her target.
numbers of hits and
a Use this result to estimate her chances of dividing by the total
hitting the target. number of shots.
b On the next day in the final round of the
competition she scored 105 hits out of 120
shots. Obtain the ‘best’ estimate of her
hitting the target with any shot.
Page 188 of 525
ESTIMATING PROBABILITIES (Chapter 15) 319

4 Paulo catches a 7:45 am tram to school. During a period of 79 days he arrives at school
on time on 53 occasions. Estimate the probability Paulo:
a arrives on time b arrives late.
5 A circular target has 8 sectors numbered 1 to 8. The sectors all have angle 45o . The
target rotates about its centre at high speed. When Max throws a dart into it 120 times,
the dart hits the ‘8’ sector 17 times. Find the experimental probability of hitting the ‘8’
sector.
6 Don threw a tin can into the air 180 times. From these trials
it landed on its side 137 times. Later that afternoon he threw
the same tin can into the air 150 more times. It landed on its
side 92 times.
a Find the experimental probability of ‘landing on the side’
for both sets of trials. side end
b List possible reasons for the differences in the results.

B PROBABILITIES FROM TABLED DATA


When data is collected, it is often summarised and displayed in a frequency table. We can
use such a table to help calculate probabilities.

Example 2 Self Tutor


A marketing company surveys 50 Brand Frequency Relative Frequency
randomly selected people to discover Shine 10 0:20
what brand of toothpaste they use. Starbright 14 0:28
The results are in the table:
Brite 4 0:08
Clean 12 0:24
No Name 10 0:20

a Find the experimental probability that a community member uses:


i Starbright ii Clean.
b Would you classify the estimate of a to be very good, good, or poor? Why?

a i Experimental P(Starbright) ¼ 0:28


ii Experimental P(Clean) ¼ 0:24
b Poor, as the sample size is very small.

EXERCISE 15B
1 A street vendor is selling toffee apples. Some apples are Colour Freq. Rel. Freq.
red and others are green. On one day 247 toffee apples
Red 149
are sold.
Green
a Copy and complete the table. Total
b Estimate the probability that the next customer buys:
i a red toffee apple ii a green toffee apple.
Page 189 of 525
320 ESTIMATING PROBABILITIES (Chapter 15)

2 Three types of tickets are available for a concert. Type Freq. Rel. Freq.
These are Adult, Pensioner and Child. All tickets Adult 238
were sold and a summary appears in the table. Pensioner 143
a Copy and complete the table given. Child 119
b How many seats does the theatre have? Total
c If the seats are numbered and one number is
chosen at random, find the probability that the person sitting at that seat is a child.
d Is this an experimental probability?
3 A supermarket sells four fragrances of air Fragrance Freq. Rel. Freq.
freshener. During a period of a week it sells Rose 59
the numbers shown in the table opposite. Violet 72
a Copy and complete the table. Lavender 81
b Estimate the probability that the next Woodland Breeze 104
customer will buy: Total
i Violet
ii Rose or Woodland Breeze.
4 A small island country imports four models of cars which Model Freq. Rel. Freq.
we will call A, B, C and D. A 0:25
a Copy and complete the given table. B 0:35
b Estimate the probability that the next car which C 40
passes the town hall is model: D 0:275
i C ii C or D. Total

C PROBABILITIES
FROM TWO WAY TABLES
Sometimes data is categorised by not only one, but two variables, and the data is represented
in a two way table. We can estimate probabilities from a two way table just as we do from
a regular frequency table.

Example 3 Self Tutor


60 students were randomly selected and asked Physics
whether they studied Physics and English. Yes No Total
a Copy and complete the table by finding the Yes 15 12
total of each row and column. English
No 13 20
b What does the 13 in the table represent? Total
c Estimate the probability that a randomly
selected student who studies English also studies Physics.
d Estimate the probability that a randomly selected student studies English but not
Physics.

Page 190 of 525


ESTIMATING PROBABILITIES (Chapter 15) 321

a Physics b 13 of the selected students study


Yes No Total Physics but do not study English.
Yes 15 12 27
English
No 13 20 33
Total 28 32 60
c 27 students study English, 15 of whom also study Physics.
) P(randomly selected student who studies English also studies Physics) ¼ 15
27
¼ 0:556
d 12 out of the 60 students study English but not Physics.
12
) P(randomly selected student studies English but not Physics) ¼ 60 ¼ 0:2

Since the data only represents a sample of the population, the results are only estimates of
the true probabilities. The larger the size of the sample, the more accurate the estimates will
be.

EXERCISE 15C
1 Students at a high school were surveyed to Instrument
determine whether they played a sport or a musical Yes No Total
instrument. Yes 19 25
Sport
a Copy and complete the table alongside. No 11 14
b Estimate the probability that a randomly chosen Total
student:
i who plays sport does not play an instrument
ii plays an instrument.

2 A sample of adults were asked whether they Gender


had a driver’s licence. The results were Male Female Total
further categorised by gender as shown: Yes 62 17
a Copy and complete the table alongside. Licence No 8 3
b Estimate the probability that a randomly Total
chosen:
i male has a driver’s licence ii female has a driver’s licence.
c Which of the estimates in b is more likely to be accurate? Explain your answer.
3 At a supermarket, a taste-testing stall was set up to Said
determine whether people could taste the difference A B Total
between two brands of cola, A and B. 100 randomly A 38 12
selected shoppers were given either A or B, then Given
asked to say what they were drinking. B 17 33
Total
a Copy and complete the table.
b Estimate the probability that a randomly selected shopper who is given:
i A could correctly identify their drink
ii B could correctly identify their drink.
Page 191 of 525
322 ESTIMATING PROBABILITIES (Chapter 15)

4 100 randomly chosen members of Vote


an electorate are asked whether they Yes Undecided No Total
will vote for the current mayor in under 30 15 19 14
the upcoming election. The results Age over 30 26 16 10
are categorised by age.
Total
a Copy and complete the table.
b Estimate the probability that a randomly chosen person:
i over 30 has decided to vote for the current mayor
ii has decided not to vote for the current mayor
iii is over 30 and is undecided
iv under 30 has decided whether or not to vote for the current mayor.

D CHANCE INVESTIGATIONS
When we perform experiments, the relative frequency of an outcome gives us an estimate
for the probability of that outcome. The greater the number of trials, the more we can rely
on our estimate of the probability.
The most commonly used equipment for experimental probability and games of chance are
described below:

COINS
When a coin is tossed there are two possible sides that could show upwards: the head which
is usually the head of a monarch, president, or leader, and the tail which is the other side
of the coin. We expect a head (H) and a tail (T) to have equal chance of occurring, so we
expect each to occur 50% of the time. So,
the probability of obtaining a head is 12 , and

the probability of obtaining a tail is 12 :

The table below shows actual experimental results obtained for tossing a coin:

Number of tosses H T %H %T
10 7 3 70:0 30:0
100 56 44 56:0 44:0
1000 491 509 49:1 50:9

each is nearly 50%

These experimental results support our expectations and suggest the general rule:
“The more times we repeat an experiment, the closer the results will be
to the theoretical results we expect to obtain.”
Page 192 of 525
100

100

100

100
50

50

50

50
0

0
25

75

95

25

75

95

25

75

95

25

75

95
5

IB MYP_3
cyan magenta yellow black

Y:\HAESE\IB_MYP3\IB_MYP3_15\322IB_MYP3_15.CDR Monday, 26 May 2008 1:44:57 PM PETER


ESTIMATING PROBABILITIES (Chapter 15) 323

DICE
Dice is the plural of die.
The most commonly used dice are small cubes with the
numbers 1, 2, 3, 4, 5 and 6 marked on them using dots. The
numbers on the face of a cubic die are arranged such that
the sum of each pair of opposite faces is seven.

SPINNERS
A simple spinner consists of a regular polygon or sometimes a circle
with equal sectors. We place a toothpick or match through its centre.
Alongside is a square spinner. It shows a result of 1 since it has come to
rest on the side marked 1.
A spinner such as that alongside may be used instead of a die,
providing all angles are exactly 60o .
The result shown is 2, since the pointer came to rest on the sector
marked 2.
The roulette wheel is an example of a much more
sophisticated spinner.

©iStockPhoto/Craig Veltri

PLAYING CARDS Suit: clubs


King Queen
A standard deck of playing cards consists of 52 Ace
cards. There are 4 suits: hearts and diamonds
(red), and spades and clubs (black).
Each suit contains 13 cards with face values
Suit:
Ace, 2, 3, 4, 5, 6, 7, 8, 9, 10, Jack, Queen, spades
King.
Jacks, Queens and Kings are all called ‘picture’ Value of
cards. card

In the following investigations you use technology to run simulations of experiments. The
use of technology enables us to gather vast numbers of results almost instantaneously, and in
a form which is easy to recognise and interpret.

INVESTIGATION 1 ROLLING A PAIR OF DICE


You will need: 2 normal six-sided dice with
numbers 1 to 6 on the faces.

Page 193 of 525


100

100

100

100
50

50

50

50
0

0
25

75

95

25

75

95

25

75

95

25

75

95
5

IB MYP_3
cyan magenta yellow black

Y:\HAESE\IB_MYP3\IB_MYP3_15\323IB_MYP3_15.CDR Monday, 26 May 2008 3:11:29 PM PETER


324 ESTIMATING PROBABILITIES (Chapter 15)

What to do:

1 The illustration shows that when two dice are rolled, there are 36 possible outcomes.
Of these, f1, 3g, f2, 2g and f3, 1g give a sum of 4.
Copy and complete the table of expected or theoretical results:

Sum 2 3 4 5 ¢¢¢ 12
3
Fraction of total 36
Fraction as decimal 0:083

2 If a pair of dice is rolled 360 times, how many of each result (2, 3, 4, ...., 12) would
you expect to get? Extend your table by adding another row. Write your expected
frequencies within it.

3 Toss the two dice 360 times. Record the sum of the two numbers for each toss using
a table.
WORKSHEET
Sum Tally Frequency Relative Frequency
2
3
4
5
..
.
12
Total 360 1

4 Pool as much data as you can with other students. Find the overall relative frequency
of each sum.

5 Use the two dice simulation from the computer package on the CD to SIMULATION
roll the pair of dice 10 000 times. Repeat this 10 times and on each
occasion record your results in a table like that in 3. Are your results
consistent with your expectations?

Page 194 of 525


ESTIMATING PROBABILITIES (Chapter 15) 325

INVESTIGATION 2 BLACKJACK
Blackjack is a game which uses playing cards. It is most commonly played
in casinos.
In blackjack, each playing card is assigned a points value:
² cards 2 through to 10 are worth their face value, so 2s are worth 2
points, 3s are worth 3 points, and so on
² Jacks, Queens and Kings are worth 10 points
² Aces are worth 1 or 11 points, depending on which is better for the
player.
Players are initially dealt two cards. They must then decide whether to accept additional
cards in an attempt to obtain a total points value as close as possible to 21, without going
over.
We will now find experimental probabilities for obtaining the various point scores possible
from the initial 2 card deal.
What to do:

1 Shuffle a pack of cards, and draw the top two cards from the deck.
2 Calculate the sum of the points value of these two cards.
For example,
= 5 + 8 = 13 = 10 + 4 = 14

= 11 + 10 = 21
Count aces as 11,
3 Return the cards to the pack. except for 2 aces which
is 11 + 1 = 12 since
4 Do this 200 times, recording your results in a table: 11 + 11 = 22 and
this is more than 21.
Sum Tally Frequency Relative Frequency
4
5
6
..
.
21
Total 200 1

5 From your data, which point score occurs most frequently after the two card deal?

6 Use the simulation to repeat this process 10 000 times. Compare your SIMULATION
results with those of the simulation.

Page 195 of 525


326 ESTIMATING PROBABILITIES (Chapter 15)

INVESTIGATION 3 CONCEALED NUMBER TICKETS


Many clubs have ticket machines which contain sets of consecutive numbers
from 0001, 0002, 0003, ... up to 2000. These games are relatively
inexpensive to play and are used as fund-raisers for the club.
Tickets are ejected from the machine at random at a cost of 50 cents each.
A small cardboard cover is removed to reveal the concealed number.
Suppose a golf club can buy golf balls as prizes Winning Numbers Prize
for $5:50 each and a set of 2000 tickets for the
777 4 golf balls
machine at $50. The club shows the following
winners table: 1000, 2000 2 golf balls
What to do: any multiple of 25 1 golf ball

1 If all tickets are sold, how many balls are paid out as prizes?
2 Determine the total cost to the club for a complete round of 2000 tickets going
through the machine.
3 What profit is made by the club?
4 If you purchase one ticket, what is your chance of winning at least one ball?

INVESTIGATION 4 FURTHER SIMULATIONS


A INSTANT MONEY AND BINGO
WORKSHEET
Click on the icon to access this investigation.

B DICE GOLF
Over a long period of time, Joe noticed that on ‘par 3’ golf holes he scores
a two, a three, a four or a five in the ratio 1 : 2 : 2 : 1:
Simulate Joe’s golf scoring for par 3 holes using a die which has two on one face, three
on two faces, four on two faces and five on one face.
What to do:
1 Roll the die 100 times to simulate Joe’s next 100 par 3 hole scores.
2 Find Joe’s average score per hole for the results in 1.
1 2 2 1
3 a Find 6 £2 + 6 £3 + 6 £4 + 6 £ 5.
b Explain the significance of this calculation.
c How is this calculation related to 2?

C MATHEMATICS COMPETITION
Click on the icon. This simulation tests your ability to select correct answers SIMULATION
by guessing or by chance alone. Run the simulation several times.

Page 196 of 525


ESTIMATING PROBABILITIES (Chapter 15) 327

REVIEW SET 15A


1 A cube has 2 faces painted green and 4 faces painted red. When it is tossed
97 times, the uppermost face is green on 35 occasions. Find the experimental
probability of the uppermost face being red.

2 In a large country hospital, 72 girls and 61 boys were born in a month.


a How many children were born in total?
b Estimate the probability that the next baby born at this hospital will be:
i a girl ii a boy.

3 A survey of three-child families was Result Frequency Rel. Freq.


conducted and the following results 3 boys 21
obtained:
2 boys, 1 girl 62
a How many families were surveyed? 1 boy, 2 girls 55
b Copy and complete the table. 3 girls 18
c Estimate the chance of a randomly Total
selected 3-child family containing:
i 2 boys and 1 girl ii 3 girls.

4 The year 8 students in a school were all Test


given a maths test. After the test a sample Pass Fail Total
of students were asked whether they studied Yes 21 7
for the test and whether they passed or Studied No 12 8
failed.
Total
a Copy and complete the table.
b If a year 8 student is chosen at random, estimate the probability that he or she:
i passed the test ii did not study and failed the test.
c If a student who studied for the test is selected at random, estimate the probability
that he or she passed the test.

REVIEW SET 15B


1 A spinner is made using a regular pentagon,
as shown. When the spinner was twirled 378
times it finished on red 72 times. Find the
experimental probability of spinning a red.

2 Two coins are tossed many times and the Result Frequency Rel. Freq.
number of heads recorded each time. The 2 heads 78
results are given in the table. 1 head 149
a Copy and complete the table. 0 heads 73
b Use the data to estimate the probability Total
that the result of the next toss will be:
i 2 heads ii at least 1 head.

Page 197 of 525


328 ESTIMATING PROBABILITIES (Chapter 15)

3 A marketing company was commissioned to investigate the types of cereal people


preferred for breakfast. The results of the survey are shown below:
a How many people were randomly selected Brand Freq. Rel. Freq.
in this survey?
Wheat Bites 198
b Calculate the relative frequency of each
Corn Crisps 115
brand of cereal.
c Using the results of the survey, estimate Rice Flakes 110
the probability that a randomly selected Bran Snacks 77
person: Total
i prefers Wheat Bites
ii does not prefer Bran Snacks?

4 The table shows the number of students in Fair hair Dark hair Total
a school with fair hair and dark hair.
Boys 418 577
a Copy and complete the table.
Girls 327 638
b How many students are there:
Total
i who are boys
ii who have dark hair iii in the school?
c If a student is randomly chosen from this school, what is the probability that the
student:
i is a girl ii has dark hair
iii has fair hair, if we know that the student is a girl?

KARL FRIEDRICH GAUSS (1777-1855)


Karl Friedrich Gauss is one of the greatest
mathematicians ever. As a young student he was
interested in the study of languages and was in fact fluent
in German, Latin, Greek, Sanskit, Russian and English.
When he was only 3 years old he is believed to have
corrected the wages of a team of bricklayers as calculated
by his father.
It is written that Karl’s mathematics teacher loved to set
ridiculously long arithmetic sums as impositions for his
students who misbehaved. On one occasion Karl and the
other naughy boys were set the task of adding a long
string of consecutive large whole numbers. Karl devised
a simple method which enabled him to calculate such a sum within a few seconds.
His method for the sum 1 + 2 + 3 + 4 + 5 + 6 + :::::: + 98 + 99 + 100, say, was to add
the numbers in pairs. That is,
(1 + 99) + (2 + 98) + (3 + 97) + :::::: + (49 + 51) + 50 + 100
= 49 £ 100 + 100 + 50
= 5050
Needless to say, his teacher had to find other forms of punishment.
Page 198 of 525
100

100

100

100
50

50

50

50
0

0
25

75

95

25

75

95

25

75

95

25

75

95
5

IB MYP_3
cyan magenta yellow black

Y:\HAESE\IB_MYP3\IB_MYP3_15\328IB_MYP3_15.CDR Thursday, 5 June 2008 2:21:25 PM PETER


Chapter
22
Theoretical
probability

Contents: A Sample space


B Theoretical probability
C Using grids to find probabilities
D Multiplying probabilities
E Using tree diagrams
F Expectation
G Odds

Page 199 of 525


100

100

100

100
50

50

50

50
0

0
25

75

95

25

75

95

25

75

95

25

75

95
5

IB MYP_3
cyan magenta yellow black

Y:\HAESE\IB_MYP3\IB_MYP3_22\437IB_MYP3_22.CDR Friday, 30 May 2008 2:51:50 PM PETER


438 THEORETICAL PROBABILITY (Chapter 22)

OPENING PROBLEM
Dieter has noticed that when he is driving, he is stopped by one in every
two sets of traffic lights. We say the probability that Dieter has to stop at
traffic lights at any given intersection is 12 .
Things to think about:
² If Dieter has to pass through two consecutive intersections with traffic lights, can
you calculate the probability that he will have to stop?
² What do you think will be the probability that Dieter has to stop if he passes through
four consecutive intersections with traffic lights?
² Do you think that this is realistic? What other factors should you consider?

A SAMPLE SPACE
The outcomes of an experiment are the different possible results we could obtain in one
trial.
The sample space of an experiment is the set of its possible outcomes.

Suppose we have 10 cards with the numbers 1 to 10 written on them. If we take a card at
random, it could have any of the numbers from 1 to 10 on it.
The outcomes are the numbers 1, 2, 3, 4, 5, 6, 7, 8, 9 or 10.
So, the sample space is f1, 2, 3, 4, 5, 6, 7, 8, 9, 10g.
The number of elements in the sample space is 10.

An event is one or more of the outcomes of an experiment that have a particular


characteristic.
In the example above, an event E could be ‘getting a prime number’.
The outcomes in the event are 2, 3, 5 and 7. The number of outcomes in the event is 4.

EXERCISE 22A
1 List the sample space for:
a taking a ticket at random from a box containing red and blue tickets
b rolling a 6 sided die
c taking a card at random from a pack of playing cards and looking at its suit
d twirling a spinner with 6 segments marked A, B, C, D, E, and F.
e choosing at random a day of the week
f the results of a student sitting a test.
2 We have 12 cards which have the numbers from 1 to 12 written on them. A card is taken
at random.
a Find the number of elements in the sample space.
Page 200 of 525
THEORETICAL PROBABILITY (Chapter 22) 439

b Event A is ‘getting an even number’.


i List the outcomes in the event.
ii Give the number of outcomes in the event.
c Event B is ‘getting a number less than 5’.
i List the outcomes in the event.
ii Give the number of outcomes in the event.
d Event C is ‘getting a multiple of 4’.
i List the outcomes in the event.
ii Give the number of outcomes in the event.
3 List the sample space for the genders of a litter of two puppies. Let MF represent ‘the
first is male and the second is female’.

B THEORETICAL PROBABILITY
Consider the octagonal spinner shown.
Since the spinner is symmetrical, when it is spun the arrowed marker
could finish with equal likelihood on each of the sections marked 1 to 8.
We therefore say that the likelihood of obtaining a particular number, for
example 4, is
1
1 chance in 8, 8, 12 12 % or 0:125 :
This is a mathematical or theoretical probability and is based on what we theoretically
expect to occur.

The theoretical probability of a particular event is the theoretical chance of that event
occurring in any trial of the experiment.
We read 38 as ‘3
Consider the event of getting ‘6 or more’ from one spin of the octagonal
chances in 8’.
spinner. There are three favourable outcomes (6, 7 or 8) out of the eight
possible outcomes, and each of these is equally likely to occur.
3
So, P(6 or more) = 8

In general, for an event E containing equally likely possible results:

the number of outcomes in the event E


P(E) = .
the total number of possible outcomes

Example 1 Self Tutor


A ticket is randomly selected from a basket containing 3 green, 4 yellow and 5 blue
tickets. Determine the probability of getting:
a a green ticket b a green or yellow ticket
c an orange ticket d a green, yellow or blue ticket

Page 201 of 525


440 THEORETICAL PROBABILITY (Chapter 22)

The sample space is fG1 , G2 , G3 , Y1 , Y2 , Y3 , Y4 , B1 , B2 , B3 , B4 , B5 g


which has 3 + 4 + 5 = 12 outcomes.
a P(green) b P(a green or a yellow)
3 3+4
= 12 = 12
1 7
= 4 = 12

c P(orange) d P(green, yellow or blue)


0 3+4+5
= 12 = 12
12
=0 = 12
=1

In Example 1 notice that:


² in c an orange result cannot occur. The calculated probability is 0, which fits the fact
that it has no chance of occurring.
² in d that a green, yellow or blue result is certain to occur. It is 100% likely or certain
which is perfectly described using a 1.

For any event E, 0 6 P(E) 6 1.

COMPLEMENTARY EVENTS

If E is an event, then E 0 is the complementary event of E, and


P(E) + P(E 0 ) = 1.

A useful rearrangement is: P(E not occurring) = 1 ¡ P(E occurring)

Example 2 Self Tutor


An ordinary 6-sided die is rolled once. Determine the chance of:
a getting a 6 b not getting a 6
c getting a 1 or 2 d not getting a 1 or 2

The sample space of possible outcomes is f1, 2, 3, 4, 5, 6g.


a P(6) b P(not getting a 6)
= 1
6
= P(1, 2, 3, 4 or 5)
5
= 6

c P(1 or 2) d P(not getting a 1 or 2)


= 2
6
= P(3, 4, 5 or 6)
4
= 6

Page 202 of 525


THEORETICAL PROBABILITY (Chapter 22) 441

In Example 2, notice that P(6) + P(not getting a 6) = 1


and that P(1 or 2) + P(not getting a 1 or 2) = 1.
This is no surprise as getting a 6 and not getting a 6 are complementary events where one
of them must occur.

EXERCISE 22B
1 A marble is randomly selected from a box containing 5 green, 3 red and 7 blue marbles.
Determine the probability that the marble is:
a red b green c blue
d not red e neither green nor blue f green or red.
2 A carton contains eight brown and four white eggs.
Find the probability that an egg selected at random
is:
a brown b white.

3 In a class of 32 students, eight have one first name, nineteen have two first names, and
five have three first names. A student is selected at random. Determine the probability
that the student has:
a no first name b one first name c two first names d three first names.
4 An ordinary six-sided die is rolled once. Determine the chance of getting:
a a5 b an odd number
c a number greater than 1 d a multiple of 2.
5 In a club newsletter, 8 pages contain reports, 3 pages contain articles, and 5 pages contain
advertising. The newsletter is opened to a page at random.
a Determine the probability that it is:
i a report ii advertising
iii not advertising iv a report or articles.
b Which of the events listed in a are complementary?
6 A disc is randomly selected from a box containing two discs marked 1, three discs
marked 2, and five discs marked 3.
a Determine the probability that the chosen disc will be marked:
i 1 ii with an odd number.
b State the event that is complementary to ‘a disc with an odd number’, and determine
its probability.
7 A tennis club has 18 adult members, 13 teenage
members and 19 junior members. The name of each
member is written on a small card and placed in a box.
One card is randomly chosen from the box. What is
the probability that the member is:
a a teenager b an adult
c a junior or a teenager d a teenager and a junior?
Page 203 of 525
442 THEORETICAL PROBABILITY (Chapter 22)

C USING GRIDS TO FIND PROBABILITIES


When an experiment involves more than one operation we can still list the sample space.
If we have two operations we can use a two-dimensional grid to illustrate the sample space
efficiently.
We can use the grid to count favourable outcomes and so calculate probabilities.
This grid shows the outcomes when two coins A and B are tossed.
coin B This point represents ‘a tail from coin A’ and ‘a tail from coin B’.
T This point represents ‘a tail from coin A’ and ‘a head from coin B’.
H There are four members of the sample space.
H T coin A They are: HH, HT, TH, and TT.

Example 3 Self Tutor


Use a two-dimensional grid to illustrate the sample space for tossing a coin and rolling
a die simultaneously. From this grid determine the probability of:
a tossing a head b rolling a 2
c getting a tail and a 5 d getting a tail or a 5.

There are 12 members in the sample space.


coin
6 1
a P(head) = 12 = 2
T
2 1
b P(2) = 12 = 6
H
1
die c P(tail and a ‘5’) = 12
1 2 3 4 5 6
7
d P(tail or a ‘5’) = 12 fthe enclosed pointsg

Example 4 Self Tutor


Two square spinners, each with 1, 2, 3 and 4 on their edges, are twirled simultaneously.
Draw a two-dimensional grid of the possible outcomes.
Use your grid to determine the probability of getting:
a a 3 with each spinner b a 3 and a 1 c an even result with each spinner.

spinner 2 The sample space has 16 members.


1
4 a P(a 3 with each spinner) = 16
2
3 b P(a 3 and a 1) = 16 fcrossed pointsg
2 1
= 8
1
c P(an even result for each spinner)
spinner 1 4
1 2 3 4 = 16 = 14 fcircled pointsg

Page 204 of 525


THEORETICAL PROBABILITY (Chapter 22) 443

EXERCISE 22C
1 Draw the grid of the sample space when a $1 and a $2 coin are tossed simultaneously.
Hence determine the probability of getting:
a two heads b two tails
c exactly one head d at least one head.
2 Bag A contains one red disc and one blue disc. Bag B
contains one red, one blue, and one white disc. Draw a grid
of the sample space when one disc is taken at random from
each bag. Hence determine the probabilities of getting:
a two red discs b two discs the same colour A B

c a white disc d two discs that are different colours.

3 a Use a grid to illustrate the sample space when a coin is tossed


and a spinner with 3¡equal sectors A, B, and C is twirled.
b i List the sample space.
ii How many outcomes are possible?
c Use your grid to determine the chance of getting:
i an A and a head ii a head and not an A
iii an A iv an A or a B and a tail.

4 a Draw a grid to represent the sample space when


two spinners each with 3 equal sectors 1, 2, and 3
are twirled simultaneously.
b List the sample space.
c How many outcomes are possible?
d Use your grid to determine the chance of getting:
i two 2s ii two odd numbers
iii a 2 and a 3 iv two numbers with a sum of 3
v two numbers the same vi two numbers with a sum of 3 or 5.
5 A coin is tossed and a spinner with six equal sectors marked
1, 2, 3, 4, 5 and 6 is twirled.
a Draw a grid of the sample space.
b How many outcomes are possible?
c Use your grid to determine the chance of getting:
i a tail and a 3
ii a head and an even number
iii a tail and a number greater than 4
iv a 6
v no 6s
vi any number except 5 and a head
vii a head or a 6.

Page 205 of 525


444 THEORETICAL PROBABILITY (Chapter 22)

D MULTIPLYING PROBABILITIES
In the previous section we used two-dimensional grids to represent sample spaces and hence
find answers to certain types of probability problems.
coin
Consider tossing a coin and rolling a die simultaneously.
T
What is the probability of getting a head and a 5?
H
From the grid there are 12 possible outcomes but only
one with the property that we want. So, the probability die
1 2 3 4 5 6
1
is 12 .
1 1 1 1 1
But P(a head) = 2 and P(a ‘5’) = 6 and 2 £ 6 = 12 .

This suggests that P(a head and a ‘5’) = P(a head) £ P(‘5’), so we multiply the separate
probabilities.

In general: If A and B are two events then P(A and B) = P(A) £ P(B).

Example 5 Self Tutor


Every Sunday morning Dagma and Karl go fishing. Karl says that he catches fish 9
mornings out of 10, and Dagma says she catches fish 4 mornings out of 5. If you can
believe what they say, determine the probability that:
a Dagma and Karl both caught fish last Sunday
b only Dagma caught fish last Sunday c neither of them caught fish last Sunday.

9
P(Karl catches fish) = 10
9 1
) P(Karl does not catch fish) = 1 ¡ 10 = 10
P(Dagma catches fish) = 45
4 1
) P(Dagma does not catch fish) = 1 ¡ 5 = 5

a P(both caught fish)


= P(Dagma and Karl caught fish)
= P(Dagma caught fish) £ P(Karl caught fish)
4 9 18
= 5 £ 10 = 25
b P(only Dagma caught fish)
= P(Dagma caught fish and Karl did not catch fish)
= P(Dagma caught fish) £ P(Karl did not catch fish)
4 1 2
= 5 £ 10 = 25
c P(neither caught fish)
= P(Dagma did not catch fish and ¡Karl did not catch fish)
= P(Dagma did not catch fish) £ P(Karl did not catch fish)
1 1 1
= 5 £ 10 = 50

Page 206 of 525


THEORETICAL PROBABILITY (Chapter 22) 445

EXERCISE 22D
1 Irena and Natasha are young ice skaters. They are learning to do a triple loop. Irena falls
on average half the time when she attempts this jump, while Natasha falls on average
once in every three times. If they each have one attempt at the triple loop, determine the
probability that:
a they are both successful b they both fall c Natasha succeeds but Irena falls.
2 Carlos does his shopping at the same time each Friday. He estimates that the probability
that he has to queue at the supermarket checkout is 56 , and the probability that he has
to queue at the post office is 14 . When he does his shopping next Friday, what is the
probability that he will queue at: a both places b neither place?
3 Helena has an 80% chance and Rodriquez has a 50% chance of obtaining ‘A’ grades in
their History examination. Find the probability that:
a both will obtain ‘A’ grades b Rodriquez will get an ‘A’ and Helena will not
c neither will obtain ‘A’ grades.
4 Nigel fires 500 arrows at a target and hits the target on 475 occasions. Use this
information to estimate the probability that he will hit the target with every shot when
he fires: a one arrow b two arrows c three arrows.
1
5 The probability of a woman giving birth to identical twins is approximately 250 .
Find the probability that a woman will have two sets of identical twins with her first
two births.

E USING TREE DIAGRAMS


Tree diagrams can be used to illustrate sample spaces provided that the alternatives are not
too numerous.
Once the sample space is illustrated, the tree diagram can be used for determining probabilities.
Consider Example 5 again. The tree diagram for this information is:
F = caught fish
N = did not catch fish Karl’s Dagma’s
result result
4
5 F
9
10
´ 54 = 36
50
9 F
10 1 N
9
10
´ 15 = 9
50
5
4
1 5 F
1
10
´ 54 = 4
50
10 N
1 N
1
10
´ 15 = 1
50
5
Notice that:
² the probabilities for catching and not catching fish are marked on the branches
² there are four alternative paths and each path shows a particular outcome
² all outcomes are represented and the probabilities are obtained by multiplying the
probabilities along each branch
² the probability of an event containing two or more outcomes is obtained by adding the
probabilities of each of the outcomes.
Page 207 of 525
446 THEORETICAL PROBABILITY (Chapter 22)

Example 6 Self Tutor


During the summer holidays, the probability that Hiroko plays tennis on any day is 57 ,
and the probability that she swims on any day is 35 .
a Draw a tree diagram to illustrate this situation.
b Use the tree diagram to determine the chance that on any day Hiroko:
i plays tennis and swims ii swims but does not play tennis.

a Let T represent the event ‘Hiroko plays tennis’ and S represent the event ‘Hiroko
swims’.
) P(T ) = 57 and P(T 0 ) = 27 , also P(S) = 35 and P(S 0 ) = 25 .
swimming Outcome Probability
tennis
_Et 5 3 15
S T and S 7 £ 5 = 35 X
Tu_ T
Wt_ S' T and S 0 5
7 £ 2
5 = 10
35
0 2 3 6
Wu_ Et_ S T and S 7 £ 5 = 35 ¤
T'
0 0 2 2 4
Wt_ S' T and S 7 £ 5 = 35

b i P(plays tennis and swims) ii P(swims but does not play tennis)
= P(T and S) X = P(S and T 0 ) ¤
5 3 2 3
= 7 £ 5 = 7 £ 5
3 6
= 7 = 35

Example 7 Self Tutor


Bag A contains three red and two yellow tickets. Bag B contains one red and four
yellow tickets. A bag is randomly selected by tossing a coin, and one ticket is removed
from it. Determine the probability that it is yellow.

ticket outcome
bag 3–
Bag A Bag B 5 R A and R To get a yellow
–1 A we take either the
2 2– Y A and Y X first branch
5
3R 1R 1– marked with a tick
2Y 4Y –1
5 R B and R or the second one
2 B marked with a tick
4– Y B and Y X
5 and add the
P(yellow) = P(A and Y) + P(B and Y) probabilities.
1 2 1 4
= 2 £ 5 + 2 £ 5 fbranches marked with a Xg
6
= 10
3
= 5

Page 208 of 525


THEORETICAL PROBABILITY (Chapter 22) 447

EXERCISE 22E
1 Suppose this spinner is spun twice.

2nd spin
1st spin
B

a Copy and complete the branches on the


tree diagram shown.

b Find the probability that blue appears on both spins.


c Find the probability that white appears on both spins.
d Find the probability that different colours appear on the two spins.
e Find the probability that the same colour appears on the two spins.
f Find the probability that red appears on exactly one of the spins.

2 A die has four faces marked A and two faces 1st roll 2nd roll
marked B. A is the event ‘the uppermost face is
A’, and B is the event ‘the uppermost face is B’. A
a Determine: i P(A) ii P(B).
b Copy and complete the tree diagram for two B
rolls of the die.
c Using the tree diagram, determine the probability that:
i both results are A
ii both results are the same letter
iii the first roll is B and the second roll is A
iv the result is two different letters.

3 The weather forecast gives a 20% chance of


rain tomorrow. If it is fine, the probability that
Bernard will go sailing is 80%, but if it rains, the
probability that he will go sailing is only 10%.
Display the sample space of possible outcomes
on a tree diagram.
Determine the probability that tomorrow:
goes
a is fine and Bernard goes sailing sailing
b is wet and Bernard does not go sailing fine
c Bernard goes sailing.

doesn’t go
sailing

Page 209 of 525


448 THEORETICAL PROBABILITY (Chapter 22)

4 When Dieter passes through an intersection with traffic lights, the probability that he has
to stop is 12 . If he has to pass through two consecutive intersections with traffic lights,
draw a tree diagram to illustrate the possible outcomes.
Determine the probability that Dieter: 2nd intersection
1st intersection stops
a will not have to stop
stops
b will have to stop at both sets of lights
c will have to stop at exactly one set of
lights does
d will have to stop. not stop

5 Luke and Gaston are chefs. Luke cooks 60% of the time and burns 4% of what he cooks.
Gaston cooks the remainder of the time and burns 3% of his cooking.
Determine the probability that the next good
meal: Luke
a is cooked by Gaston and is burnt burnt

b is cooked by Luke and is good


c is burnt.

6 Moira has two cartons each containing a dozen eggs.


Carton A has three white and nine brown eggs, and
carton B has eight white and four brown eggs. Moira
chooses a carton at random and randomly selects an
egg. Determine the probability that it is white.

7 Box X contains three green and two orange tickets. Box Y contains four green and six
orange tickets. A box is chosen at random by flipping a coin and one ticket is taken
from it. Determine the probability that the ticket is orange.
8 Three buckets A, B, and C, each contain some old and some new tennis balls. Bucket A
has three new and three old balls, bucket B has four new and five old balls, and bucket
C has two new and four old balls.
A bucket is selected using a triangular spinner with three equal sides marked A, B, and
C. One tennis ball is selected at random from the bucket. Determine the probability that
it is new.

F EXPECTATION
Suppose a die is rolled 120 times. On how many occasions would you expect the result to
be a “six”?
The possible outcomes when rolling a die are 1, 2, 3, 4, 5 and 6,
and each of these is equally likely to occur.
1
So, we would expect 6 of them to be a “six”.
1
Since 6 of 120 is 20, we expect 20 of the 120 rolls of the die to yield a “six”.
Page 210 of 525
100

100

100

100
50

50

50

50
0

95
25

75

95

25

75

95

25

75

95

25

75
5

IB MYP_3
cyan magenta yellow black

Y:\HAESE\IB_MYP3\IB_MYP3_22\448IB_MYP3_22.CDR Monday, 2 June 2008 3:10:32 PM PETER


THEORETICAL PROBABILITY (Chapter 22) 449

In general:

If there are n trials of an experiment and the probability of an event


occurring is p for each trial, then the expectation of the
occurrence of that event is n £ p:

Example 8 Self Tutor


Every box of Bran-Plus cereal has a 1 in 3 chance of containing a prize. In one month
Stan buys 6 boxes of Bran-Plus. How many prizes would you expect him to win?

1
p = P(box contains a prize) = 3

For a sample of n = 6 boxes, the expected


number of prizes is n£p
1
=6£ 3
=2

EXERCISE 22F
1 A pair of dice is rolled.
a What is the probability of rolling a pair of ones?
b If the dice are rolled 540 times, how many times would you expect a pair of ones
to appear?

2 a If 3 coins are tossed, what is the chance that they all fall heads?
b If the 3 coins are tossed 200 times, on how many occasions would you expect them
all to fall heads?
3 A library found that the probability of a borrowed book being returned on time is 0:68.
In one day the library lent 837 books. How many of those books can the library expect
to be returned on time?
4 In a money wheel game, the wheel shown alongside is 9 1
1
spun, and the player wins the amount in euros indicated by
3

the marker.
1

a What are your chances of spinning the wheel once and


6

winning: 1 3
1

i E9 ii E6 iii E3 iv E1?
1
b Your expected return from spinning a 9 is 12 £ E9. What is your expected return
from spinning:
i a6 ii a 3 iii a 1?
c Use b to find the total expected return from one spin of the wheel.
d The game costs E4 to play. What is your expected profit or loss after one game?

Page 211 of 525


450 THEORETICAL PROBABILITY (Chapter 22)

G ODDS
In the horse and dog racing industries, the chances
of winning (coming first) or running a place (coming
first, second or third) are quoted as odds.

Odds are usually given against a particular event occurring


and are expressed as the ratio
number of ways you can lose : number of ways you can win.

For example, if the odds of Rogue winning are 3 : 1, then since 3 + 1 = 4,


P(Rogue loses) = 34 and P(Rogue wins) = 14 .

m n
In general, if the odds are m : n, then P(losing) = , P(winning) = .
m+n m+n

Odds quoted as evens means 1 : 1 and so P(losing) = P(winning) = 12 .

Betting also uses odds.


For example, if Rogue’s odds are 3 : 1 and Rogue wins, the bookmaker pays out $3 for
every $1 bet.
So, if a punter bets $20 on Rogue to win and Rogue does win, the punter wins
$60. The bookmaker makes a payment of $80 to the punter, the $20 original
bet plus the $60 won.

Example 9 Self Tutor


Rogue is at his peak in fitness and is quoted at odds of 3 : 5.
a Determine the bookmaker’s estimated probability that Rogue will:
i lose ii win.
b If a punter bets $20 on Rogue to win, what is the punter’s situation if Rogue:
i loses ii wins?

a For odds of 3 : 5, 3 + 5 = 8
i ) P(loses) = 38 ii P(wins) = 5
8
b i If Rogue loses, the punter is down $20.
ii If Rogue wins, for every $5 bet the punter wins $3.
) the punter wins $3 £ 4 = $12.
As the punter gets back the original bet, he or she is up $12:
The return is $20 + $12 = $32:

Page 212 of 525


THEORETICAL PROBABILITY (Chapter 22) 451

EXERCISE 22G
1 Determine the bookmaker’s estimated probability of i winning ii losing when the
odds are:
a 2:1 b 5:1 c 50 : 1 d 1:1 e 2 : 3.
2 Calculate the amount won by a punter who places a bet of:
a $10 on a horse at 4 : 1 and the horse wins
b $80 on a horse at 2 : 3 and the horse wins.

3 A punter goes to a race meeting with $100. He has planned which horses to bet on in
each race, and will keep betting until he runs out of money or there are no more races.
There are 5 races at the meeting.
a Copy and complete the following table, which is the punter’s dream scenario:
Race Odds * Bet ($) Result Win/Lose ($) Money remaining ($)
100
1 2:1 20 W
2 5:4 20 W
3 10 : 1 5 W
4 Evens 30 W
5 1:4 60 W
* Odds for his chosen horse
b At the end of the last race, what is the punter’s “dream” situation?
c In reality, the results of the punter’s horses are: Lose, Win, Win, Lose, Lose.
At the end of the last race, what is the punter’s “reality” situation?
d Assuming the odds given accurately reflect the probability of each horse winning,
work out the punter’s expectation for the bet of $20 on one of the horses. Interpret
the meaning of this answer.
e Is it then reasonable to assume that the bookmaker also has zero expectation for the
bet? If so, where does his profit come from?

REVIEW SET 22A


1 Draw a two-dimensional grid representing the sample space when an octahedral or
8-sided die is rolled and a coin is tossed simultaneously. Hence, determine the
likelihood of:
a a head and an 8 b an even number and a tail
c an even number or a tail
2 A bag of mixed lollies contains 10 mints and 6 chocolate caramels. A second bag
contains 8 mints and 8 chocolate caramels. A bag is randomly chosen by tossing a
coin, and a lolly is then taken from it. Construct a tree diagram to show the sample
space and hence determine the probability that:
a a chocolate caramel will be selected b a mint will be selected.
Page 213 of 525
452 THEORETICAL PROBABILITY (Chapter 22)

3 When Sam plays Joe at squash, Sam has a probability of 47 of winning any
set they play. If they play two sets, determine the likelihood that Sam wins:
a the first set b both sets c neither set.
4 Display the possible 4-child families on a tree diagram. Determine the probability
that a randomly selected 4-child family consists of two girls and two boys.
5 Sarah has developed a new gambling game in which you roll a die. The following
payouts are made depending on the result:
1 - $1, 2 - $2, 3 - $3, 4 - $5, 5 - $10, 6 - $25.
a If you play one game, what is your expected return?
b If you play 100 games paying $10 to play each game, how much would you
expect to win or lose?

REVIEW SET 22B


1 a Illustrate on a 2-dimensional grid the sample space when a spinner with faces
A, B, C and D is spun and a die is rolled.
b Use this grid to determine the probability of getting:
i an A and a six ii an A or a six iii a B and an even number.
2 Peter has a 3 in 4 chance of successfully hitting a target and Bill has a 4 in 5 chance
of hitting the target. If they both fire simultaneously at the target, determine the
probability that:
a both hit b both miss c Peter hits and Bill misses.
3 Alec Smart and Joe Slow sit for an examination in Chemistry. Alec has a 95%
chance of passing and Joe has a 25% chance of passing. Determine the probability
that:
a both pass b both fail
c Joe passes and Alec fails d Alec passes and Joe fails
4 a Display the possible 3-child families on a tree diagram. Determine the probability
that a randomly selected 3-child family consists of two boys and a girl.
b In a survey of 200 randomly selected families with 3 children, how many would
you expect to have two boys and a girl?
5 a A punter places a bet of $20 on a horse at 5 : 1 and the horse wins. How much
does the punter win?
b What is the estimated probability that the horse wins?
6 A die has 3 red, 2 blue and one white face. When the die is rolled, a red result wins
E1, a blue result E2, and a white result E5. What is the “expected return” for one
roll of this die?

Page 214 of 525


IM3_Ch14.fm Page 384 Friday, January 4, 2008 9:29 AM

14
Probability

There is a 25% 1 in 6 people get


chance of rain skin cancer.
today.
It’s unlikely I’ll
be getting fed
around here.

Chapter Contents
14:01 Describing your chances 14:03 Theoretical probability
Investigation: Throwing dice Mathematical Terms, Diagnostic Test,
14:02 Experimental probability Revision Assignment, Working Mathematically
Reading Mathematics: Tossing a coin

Learning Outcomes
Students will be able to determine relative frequencies and theoretical probabilities.
Areas of Interaction
Approaches to Learning, Environment, Homo Faber

384
Page 215 of 525
IM3_Ch14.fm Page 385 Friday, January 4, 2008 9:29 AM

14:01 | Describing Your Chances


Language
We need to know the language of chance or probability.
certain: will always happen at random: to choose so that each
very likely: will usually happen outcome is just as likely
impossible: will never happen to happen
very unlikely: usually won’t happen probability: the calculated chance of
even chance: is just as likely to happen something happening
as it is not to happen

For each of these events, estimate its probability of happening to be: prep qu

iz
A an even chance, B less than an even chance, or C more than an even chance.
1 If I toss a coin I will get a head.
2 The next person I see driving a car will be over 20. 14:01

3 If I take a card at random from a standard pack, it will be a club.


4 If I throw a dice, I will get an even number.
5 The next child born in the hospital will be female.
6 I will see a thylacine on the way home from school.
7 If I choose a class in Year 9, at least 3 of the students will have a computer in their home.
8 If I throw two dice, their sum will be greater than 9.
9 If I throw two coins, I will have either two heads or two tails.
10 In my next maths test, I will score over 80%.

worked example
Heather is going to throw a dice.
a List all of the possible outcomes.
b Use the language at the beginning of this section to
describe the chance that:
i Heather will throw a four
ii Heather will throw a number less than four
iii Heather will throw a seven
iv Heather will throw a number less than seven.
Solution
a If a dice is thrown the possible outcomes are: 1, 2, 3, 4, 5, and 6.
b i Since only one number out of six is a four, the chance of throwing a four would be
unlikely.
ii Since three of the six numbers are less than four, there is an even chance that this
will happen.
iii There is no seven on a dice, so throwing a seven is impossible.
iv All six numbers that can be thrown are less than seven, so it is certain that the number
thrown will be less than seven.

CHAPTER 14 PROBABILITY 385


Page 216 of 525
IM3_Ch14.fm Page 386 Friday, January 4, 2008 9:29 AM

Subjective probability
• Subjective probabilities are probabilities based on
opinions or judgements. All of the facts may not
be known and we make the best prediction we can.
eg ‘There is a high probability of rain tomorrow.’
I would have more confidence in this judgement
if it were given by a meteorologist than if it were
given by my son.
• Assessments of the probability of success of Germany
or Italy as they play each other in the next World Cup
will vary greatly according to the bias of the person asked.
A more accurate assessment of the chance of success for
eachteam would come from an unbiased observer who
has studied the form of each team.
• Remember that the most likely outcome will not
always occur.

worked example
Rhonda and Alan had three girls. Rhonda was expecting
their fourth child.
‘I hope this child is a boy,’ said Alan.
‘After three girls, it’s almost sure to be a boy,’ said a friend.
Is the friend’s reasoning correct?
Solution
• For each birth, the chance of a boy will always be 1--2-
irrespective of the number of girls previously born.
So the friend’s reasoning is not correct.
• There is a chance however that the opposite is true.
Some males may have a larger percentage of sperm of one sex. The probability of having a
girl may be much higher than 1--2- . Only medical tests could reveal this.

Exercise 14:01
1 The numbers 1 to 6 were written on six counters and placed in a jar.
Two of these counters are to be chosen at random.
a List all possible outcomes of this experiment.
N u m b e rs
1 to 6

2 64 3 1
5

386 INTERNATIONAL MATHEMATICS 3


Page 217 of 525
IM3_Ch14.fm Page 387 Friday, January 4, 2008 9:29 AM

Choose a label from those on the right to answer each


question below. impossible
For the experiment of the six counters in the jar, not likely
what is the chance that:
b the numbers 0 and 1 are picked? even chance
c the numbers 5 and 6 are picked?
d two numbers less than 10 are picked? likely
e the first number selected will be even?
f the sum of the numbers will be odd? certain

6 5
10 9 6
5 5
4

B = blue
7

B R B
3

G = green
2 R = red
8

1 G G
B B B 6

2 Using the diagrams above, list all possible outcomes if:


a the spinner is spun once
b the two dice are thrown and the numbers on the upper faces added
c two balls are drawn from the container at random
d two cards are chosen at random
3 The spinner above is spun once. Choose the label to the right that Chance
best describes the probability that the number will be:
a 1 b not 8 c more than 5 impossible
d odd e 0 f less than 10
very unlikely
4 The dice above are thrown. Choose the label that best describes
the probability that the sum of the top numbers will be: unlikely
a 12 b less than 12 c less than 6
even chance
d even e 1 f less than 15
5 A coloured ball is drawn at random from the container above. likely
Which label best describes the probability that the ball is:
a blue? b green? very likely
c red? d coloured? certain
Write the events in parts a to d in order, from least likely to most likely.
6 Two cards are drawn at random from the five above. Which label best describes the probability
that the cards:
a are both fives?
b have a sum equal to 15?
c have a sum greater than 9?
d are both larger than 9?
Write the events in parts a to d in order, from least likely to most likely.

CHAPTER 14 PROBABILITY 387


Page 218 of 525
IM3_Ch14.fm Page 388 Friday, January 4, 2008 9:29 AM

7 Sri Lanka is playing cricket against England at R Premadasa Stadium, Colombo.


a From the list below, select the five factors that you think would most influence the outcome
of the match. Give a reason for each choice.
• The colour of the uniforms
• Who wins the toss
• The recent form of the players
• The number of tickets sold
• The country where it is played
• The condition of the wicket
• The weather
• The choice of umpires
• The length of the grass
• The ability of the captains
b Who do you think would win?
Give a reason for your answer.
c If Sri Lanka has never lost in Colombo,
is it possible that Sri Lanka will lose?
d Are the only possible outcomes: Sri Lanka
wins or Sri Lanka loses? Explain your answer.
8 Is each statement reasonable or not? Discuss the reasoning in each case.
a On my last four attempts, I have failed to lift this
weight. I’m sure to succeed this time.
b My diamond ring is in one of these three jewellery
boxes. I have searched two and it is not in them.
It must be in the third one.
c We almost pushed the car out of the bog that time.
Now that Chub is here to help, I’m sure we will
get it out this time.
d The first three puppies were female. The next is
sure to be male.
e I’ve turned 48 cards of this pack and have turned
only one ace. The next card I turn is sure to be
an ace.
f I’ve just heard that a fire storm has passed through
our street. I fear our home has been destroyed.
g I’m glad it’s raining today because that means it won’t rain for tennis tomorrow.
9 Discuss the contradictory nature of these statements.
a The horse was a certainty beaten.
b I was so close to winning the lottery. My number was 908 146. The winning number
was 808 146.
c The winner came from an impossible position.
10 Comment critically on the statement: ‘Since there are 26 letters in the English alphabet, the
probability that a person’s name starts with X is 1 in 26’.

388 INTERNATIONAL MATHEMATICS 3


Page 219 of 525
IM3_Ch14.fm Page 389 Friday, January 4, 2008 9:29 AM

investiga

tion
Investigation 14:01 | Throwing dice
Please use the Assessment Grid on the following page to help you understand what is required
for this Investigation 14:01
Throw two dice and record the result. Carry out this experiment 50 times.
1 Use your results to complete the table below.
Note: One result may be entered against several outcomes.
2 Choose an expression from those in the list to the right that
best describes the probability for each outcome.
Probability
Outcome Tally Frequency impossible
12 very unlikely
2 unlikely
less than 12 even chance
less than 6 likely
even very likely
1 certain
less than 10
3 Compare your findings with the answers to Exercise 14:01, question 4.

14:02 | Experimental Probability


A dice was rolled several times and the following results obtained: 5, 1, 2, 5, 6, 5, 2, 4, 6, 3 prep qu

iz
1 How many times was the dice rolled?
2 How many times was the result a 5?
3 What fraction of the throws were 5s? 14:02
4 What fraction of the throws were 2s?
5 What fraction of the throws were odd numbers?
6 A coin was tossed 10 times, resulting in 4 heads. How many tails were there?
7 A coin was tossed several times and 2--5- of the results were tails.
What fraction of the tosses were heads?
4
Simplify: 8 -----
12
- 9 16
------
20
10 24 ------
50

One way of determining the chance of something happening is by observing what occurs in a
sample ‘experiment’.
If simple equipment such as coins, dice, spinners, cards or random numbers are used to represent
real events, then the ‘experiment’ is called a simulation.

Experimental probability formula:


The experimental probability = number of times this event occurred
----------------------------------------------------------------------------------
of an event total number in sample

CHAPTER 14 PROBABILITY 389


Page 220 of 525
IM3_Ch14.fm Page 390 Friday, January 4, 2008 9:29 AM

Assessment Grid for Investigation 14:01 | Throwing dice


The following is a sample assessment grid for this investigation. You should carefully read the
criteria before beginning the investigation so that you know what is required.

Assessment Criteria (B, C, D) for this investigation Achieved ✓


1
a An organised approach has not been attempted.
Investigating Patterns

2
An organised approach has been attempted and there has 3
Criterion B

b
been some interpretation of the results. 4
An organised approach has been used and some results 5
c
and patterns have been described. 6
The correct results have been justified using a successful 7
d
strategy, with a full comparison made in question 3. 8
Little or no working out is shown and presentation 1
a
is poor. 2
Communication
Criterion C

Working out is shown and results have been recorded 3


b in a systematic way. Some interpretation of the results
is given. 4

Working out is shown and results have been recorded 5


c in a systematic way. Results are interpreted fully using
symbols and/or words. 6

1
Reflection in Mathematics

Some attempt has been made to describe the results


a
obtained. 2
Criterion D

The results obtained are described and an attempt has 3


b
been made to check them against theoretical probability. 4

Detailed justification of the results has been given, as well 5


c as a thorough comparison of theoretical and experimental
probability. 6

390 INTERNATIONAL MATHEMATICS 3


Page 221 of 525
IM3_Ch14.fm Page 391 Friday, January 4, 2008 9:29 AM

More and more statistics are being collected (empirical evidence) from which predictions can be
made. Probabilities based on this evidence are used to determine the cost of insurance, life expectancy
and the likelihood of events occurring. These estimates are often called empirical probabilities and
are a type of experimental probability. If Australia had beaten England at the SCG four of the last
five times they have played there, then it would be highly likely that Australia will win next time.
Experimental probabilities are usually based on an examination of a sample or trial run of the
activity under examination.

worked examples
1 A farmer collects 10 eggs and finds that 2 of them are bad.
If he chose another egg what is the chance of getting
another bad one?
2 The contents of 20 matchboxes were examined and the
results recorded.
Number of matches 48 49 50 51 52 53
Number of boxes 1 5 8 3 2 1
If the contents of a similar box of matches were counted, what would be the experimental
probability that it would contain 50 matches or more?
Solutions
2
1 Since 2 of the first 10 eggs were bad, it seems that ----- 10
-, ■ The probability of an
1
or --5- of the farmer’s eggs might be bad. event occurring in an
So, if the first 10 eggs were truly representative of all experiment is the same as
the farmer’s eggs, then the chance of picking another its ‘relative frequency’.
1
bad one is --5- , or 1 out of 5.
2 In the sample, 14 of the 20 boxes had 50 or more matches.
number of times this event occurred
Experimental probability = ------------------------------------------------------------------------------------------
total number in sample
= 14
------
20
In boxes like these we would expect the chance of choosing one with 50 or more matches
to be 14 7
------ or ------ .
20 10

Exercise 14:02 Foundation Worksheet 14:02


Experimental probability
1 In a class of 24, 12 were blonde, 10 were
The first 100 vehicles to pass a checkpoint gave the brunette, 2 were redheads. Find the probability
1
that one student chosen at random will be:
results in the table. If these figures truly represent the a blonde b not brunette
traffic at any time past this checkpoint, determine the
experimental probability that the next vehicle will be: Type of vehicle Frequency
a a car b a motor cycle
Cars 70
c a bus d not a car
e not a car or truck Trucks 15
Motor cycles 10
Buses 5

CHAPTER 14 PROBABILITY 391


Page 222 of 525
IM3_Ch14.fm Page 392 Friday, January 4, 2008 9:29 AM

2 We examined the contents of 37 packets of coloured


sweets. The average number of each colour in a packet Colour Frequency
is shown in the table. One sweet is taken at random Brown 10
from a new packet. Use these results to determine:
Green 5
a which colour is most likely to be picked
b which colour has the least chance of being picked Red 6
c the probability that it is red Yellow 3
Total = 24
3 A factory tested a sample of 100 light bulbs and 5 were found to be faulty. From these results,
what is the probability of buying a faulty light bulb? What is the probability of buying a
good bulb?
4 Farmer Fowler recorded the number of eggs his chickens laid
No. of eggs No. of days
each day, for 6 weeks. The results are shown in the table.
If these results are typical for Farmer Fowler’s chickens at 0 8
any time of the year find, as a fraction, the probability that 1 13
on any particular day the number of eggs laid is:
a 2 2 10
b 4 3 6
c 2 or more
Convert each of your answers to a percentage correct to 4 3
the nearest whole per cent. 5 2
Total = 42

5 A survey of 100 households was taken to determine


Brand Number
how many used certain washing powders. Based on
these results, what is the probability of a household Foam 18
chosen at random:
Suds-O 27
a using Foam brand?
b using Supersoap? Supersoap 20
c not using Pow? Pow 15
d not using any of these four brands?
6 Jenny tossed four coins 30 times and the number
of heads was recorded each time. The histogram 14
shows the results. 12
a From this experiment what is the probability 10
Frequency

that when four coins are thrown there will be: 8


i no heads?
6
ii two heads?
iii at least three heads? 4
b If this experiment were to be repeated, 2
would you expect the same results? 0
0 1 2 3 4
Number of heads

392 INTERNATIONAL MATHEMATICS 3


Page 223 of 525
IM3_Ch14.fm Page 393 Friday, January 4, 2008 9:29 AM

7 A dice was thrown 50 times and the results were recorded.


Using the results in the table: Number shown Frequency
a What is the experimental probability of throwing: 1 7
i a six?
2 5
ii a two?
iii an odd number? 3 5
b Would you expect to get seven ones every time 4 10
a dice is thrown 50 times?
5 9
6 14

4
8 20
of Grade 8 students have a shoe size greater than 10 1--2- ,
------
but 19 1
------ have a shoe size less than 11 --- . What is the chance
20 2
of a Grade 8 student having a shoe size:
a less than or equal to 10 1--2- ?
b 11 1--2- or larger?
c between 10 1--2- and 11 1--2- ?
(Note: This is every size not in a and b.)

9 a Order the following events, from least


likely to most likely.
A There will be a hail-storm tomorrow.
B The next person to visit my home
will be male.
C My next maths teacher will be over
60 years of age.
D I will see the principal of our school
next week.
E At least one member of our class
will be married within 10 years.
b Estimate the probability of each event
listed in part a, giving each on the
scale 0 to 1.

10 Explain the meaning of a probability of:


a 0 b 1--2- c 1
d 0⋅5 e 100% f 50%

CHAPTER 14 PROBABILITY 393


Page 224 of 525
IM3_Ch14.fm Page 394 Friday, January 4, 2008 9:29 AM

atic
em s
ng h

Reading mathematics 14:02 | Tossing a coin


mat

readi When tossing a coin we assume that the probability of getting


14:02 a head is 1--2- or 50%, but is this true?
Luke tossed a coin five times and graphed the percentage of
heads after each toss.
He tossed: head, head, tail, head, tail.
His graph is shown below.

100
• Percentage of heads after 3 tosses
Percentage of heads

80
= 2--3- × 100%
60 = 66 2--3- %
40 • Percentage of heads after 4 tosses
= 3--4- × 100%
20
= 75%
0
1 2 3 4 5
Number of tosses

1 Toss a coin ten times and graph the percentage of


■ • Is it reasonable to
heads after each toss. Did the percentage get closer
assume that the
to 50% as the number of tosses increased? probability is 1-2- ?
2 Would this experiment be a reasonable simulation Explain.
for the gender of babies born in a local hospital? • Does any bias exist
3 If you repeat this investigation, would you obtain in the design of this
the same graph? In what way would the second experiment?
graph resemble the first?

• I often play my son Luke at chess. How could I estimate my chances of beating him in our next game?

394 INTERNATIONAL MATHEMATICS 3


Page 225 of 525
IM3_Ch14.fm Page 395 Friday, January 4, 2008 9:29 AM

14:03 | Theoretical Probability


Simplify: prep qu

iz
1 21
------
24
2 34
------
51
3 39
------
91
4 1− 13
------
16
5 1− 27
------
30
A bag contains three white, five red and four black marbles. What fraction are:
14:03
6 white? 7 black? 8 not black?
9 What fraction of the letters of the alphabet are vowels?
10 What fraction of integers from 1 to 50 inclusive are prime?

. . . But I got 3 heads


I tossed a coin 4 when I tried that!
times and got 1 head.
That means the
probability of getting
a head must be
1 in 4!

Performing an experiment will not always give a consistent result, or even a result we may think
is most likely to occur.
In many cases we can work out the expected or theoretical probability of an event by considering
the possible outcomes. For example, when tossing a coin there are two possible outcomes, a head
or a tail.
1
Since there is only one head, the probability of throwing a head would be 1 out of 2, ie --- .
2

Drawing a picture often helps.

■ When calculating the probability of an


event we shall assume that each possible
outcome is equally likely, ie no two-headed
coins or loaded dice.

CHAPTER 14 PROBABILITY 395


Page 226 of 525
IM3_Ch14.fm Page 396 Friday, January 4, 2008 9:29 AM

worked examples
1 If a dice is rolled, what is the probability of getting:
a a six? b an odd number? c a number less than seven?
2 In a bag there are six blue marbles, four white marbles and two red marbles. What is the
probability of choosing at random:
a a blue marble? b a blue or white marble? c a pink marble?
Solutions
1 The possible outcomes when rolling a dice are 1, 2, 3, 4, 5, 6. So the number of possible
outcomes is 6.
a The number of sixes on a dice is 1. b The number of odd numbers on a dice
So the probability of throwing a is 3. So the probability of throwing an
1
six is 1 out of 6, or 6 . This can be
--
- odd number is 3 out of 6.
written as: P(odd no.) = 3--6-
P(6) = 1--6-
= 1--2-
c Since all six numbers on a dice are less
than seven, the probability of throwing ■ The probability of an
a number less than seven is: event certain to happen is 1.
P(no. < 7) = 6--6- P(sure thing) = 1
=1
2 The total number of marbles in the bag is twelve. So the number of possible outcomes is 12.
a Number of blue marbles is six. b Number of blue or white marbles is ten.
6
∴ P(blue marble) = -----
12
- ∴ P(blue or white) = 10
------
12
1 5
= ---
2
= ---
6
c Number of pink marbles is zero.
0
∴ P(pink) = -----
-
■ The probability of an event
12 that cannot happen is 0.
=0 P(impossibility) = 0

If each possible outcome is equally likely, then:


n(E)
probability of an event, P(E) = -----------
n(S)
where n(E) = number of ways the event can occur
n(S) = number of ways all events can occur
(S is used to represent the sample space, which is the set of possible outcomes.)
The probability of any event occurring must lie in the range 0  P(E)  1.

396 INTERNATIONAL MATHEMATICS 3


Page 227 of 525
IM3_Ch14.fm Page 397 Friday, January 4, 2008 9:29 AM

It must be pointed out that the probabilities of each possible event must add up to 1. As a
consequence of this, if the probability of an event occurring is P(E), then the probability of
E not occurring is 1 − P(E).

■ P(E′) = 1 − P(E)
where P(E′) is the
probability of E not
occurring.

E′ is set notation for the ‘complement’ of E, ie those outcomes outside of E. For example:
• The complementary event for getting an even number after rolling a dice is getting an odd number.
• The complementary event for drawing a red card from a deck of cards is drawing a black card.

Exercise 14:03 Foundation Worksheet 14:03


Theoretical probability
1 A coin is tossed.
A single dice is thrown. What is the probability of getting: Find the probability of:
1
a head b tail
a a one? 2 Four cards are labelled A, B, C, D.
b an even number? Find the probability of selecting the:
aA b B or C
c a number less than 3?
2 Ten coloured discs are placed in a hat. Five are red, three are yellow and two are black. If one
disc is drawn from the hat, what is the probability that the disc will be:
a red? b black? c red or black?
d not black? e blue? f red, yellow or black?
3 For each event given here, write the complementary
event.
a Getting an odd number after a dice is thrown.
b Getting a tail when a coin is tossed.
c Getting a number less than 6 when a dice is thrown.
d Drawing a spade from a standard deck of playing
• What is the
cards. complement
e Seeing red displayed on a traffic light that is of winning?
working.
f Winning a soccer match.
g Choosing a vowel from the letters of the alphabet.
4 From a standard pack of 52 playing cards, a card is
drawn at random. What is the probability that the card is:
a the Ace of diamonds? b a King?
c a red card? d a spade?
e a black Jack? f a 7 or 8?
g a picture card (Jack, Queen or King)?

CHAPTER 14 PROBABILITY 397


Page 228 of 525
IM3_Ch14.fm Page 398 Friday, January 4, 2008 9:29 AM

5 The 26 letters of the alphabet are written on cards and placed in a box. If one card is picked at
random from the box, what is the chance that the letter on it will be:
a X? b a vowel?
c M or N? d a letter in the word MATHEMATICS?
6 Stickers were placed on a dice so that the faces showed three 2s,
two 4s and a 6. If the dice is now thrown, what is the probability
that the upper surface will be:
a a 2? b a 4? c a 6?
d even? e odd? f less than 6?

7 Six students, John, Bob, Joan, Helen, Betty and Janet,


each wrote their name on a piece of paper and put it
in a hat. If one name was drawn at random, what is
the probability that the name would:
a be Helen? b start with J?
c be a girl’s name? d have four letters in it?

8 A letter is chosen at random from the word HOKKAIDO. What is the probability that the
letter is:
a O? b a consonant?
c one of the first four letters of the English alphabet? d Z?
9 The numbers from 1 to 25 are written on cards. If one card is chosen at random, what is the
probability that the number on the card will be:
a an odd number? b a multiple of 5?
c a factor of 24? d a number that contains the digit 2?
10 A five-digit number is to be formed using the digits 1, 2, 3, 4, 5. What is the probability that
the number will:
a be odd? (It ends with 1, 3 or 5.) b be even? (It ends with 2 or 4.)
c start with 5?
d be greater than 30 000? (It starts with 3 or 4.)
e be divisible by 3? (Hint: Is the sum of the digits divisible by 3?)
11 If the probability of an event is 1--5- , how many times, on average, would you expect it to occur
in 20 trials? Can you say for certain how many times it will occur?

Challenge worksheet 14:03 Probability: An unusual case

398 INTERNATIONAL MATHEMATICS 3


Page 229 of 525
IM3_Ch14.fm Page 399 Friday, January 4, 2008 9:29 AM

them
ma

atic er
Mathematical terms 14

al t
ms
complementary event random
• The opposite event that covers all • Without predetermination. 14
possibilities. • To choose ‘at random’ means that each
eg if the event is throwing a ‘6’ the outcome is equally likely to occur.
complementary event would be sample
‘not throwing at 6’. Similarly the • The list of possible outcomes.
complementary event to selecting a eg when throwing a dice the sample
girl would be selecting a boy. would be 1, 2, 3, 4, 5, or 6.
experimental probability simulation
• Determining the chance of an event • An experiment which uses simple
occurring by observing what happens in a equipment such as cards or coins to
sample experiment. represent a real event.
no. of times event occurred eg using heads or tails to represent the
exp. prob. = --------------------------------------------------------------------
total number in sample birth of boys or girls.
mutually exclusive events survey
• Events that cannot occur at the same • To gather data or information from
time. which conclusions might be drawn.
eg throwing an even number and eg to count the number of trucks in a
throwing a three with a dice. line of traffic or to question a group of
outcomes students about their favourite music.
• The possible results when calculating theoretical probability
a probability. • The probability of an event that is
eg the outcomes when tossing a coin determined by considering the possible
are heads or tails. outcomes.
probability eg the probability of throwing a ‘6’ with a
• the calculated chance of an event dice is 1--6- , because there are 6 equally
happening. likely outcomes.

Mathematical terms 14

CHAPTER 14 PROBABILITY 399


Page 230 of 525
IM3_Ch14.fm Page 400 Friday, January 4, 2008 9:29 AM

tes
stic t
diagno

Diagnostic Test 14: | Probability


• These questions reflect the important skills introduced in this chapter.
14 • Errors made will indicate areas of weakness.
• Each weakness should be treated by going back to the section listed.

Section
1 List all possible outcomes of each experiment below. 14:01
5
a A dice is thrown.
3
b A coin is tossed. 7
5
c The spinner on the right is spun.
2 The points scored by our middle school basketball team in the last 14:02
44 games is shown in the table.

My score 35–39 40–44 45–49 50–54 55–59


Frequency 3 20 14 5 2

We are about to play another game. What is the experimental probability


that our score will be:
a lower than 40?
b higher than 39?
c higher than 59?
Explain why the experimental probability that our score is higher than
59 is not the real probability.
3 In a bag there are 12 blue Lego blocks, eight white Lego blocks and four 14:03
red Lego blocks all the same size. A block is chosen at random. What is
the probability of choosing:
a a blue block?
b a blue or a white block?
c a pink block?
d anything but a white block?

400 INTERNATIONAL MATHEMATICS 3


Page 231 of 525
IM3_Ch14.fm Page 401 Friday, January 4, 2008 9:29 AM

assignm
Chapter 14 | Revision Assignment

ent
1 A dice is rolled. What is the If Huong picks a counter at random, what
probability that the uppermost is the probability that she will: 14A
face will show: a lose money?
a a 1? b neither win nor lose?
b a 0? c win money?
c a number less than 4? d not lose money?
d an even number?
5 Liam threw three darts at a dartboard
e a number divisible by 3?
20 times to gain the following scores.
2 Presuming that
Eye colour Number
the figures
shown in the Brown 110
table are typical Blue 70
of the students
in a certain Green 20
international
school, what is the probability of a person
chosen at random from this town:
a having blue eyes?
b having green eyes? 56 74 53 85 52
c not have blue eyes? 57 53 91 37 82
3 A toy box contains five red balls, four 120 100 73 45 55
green balls and a yellow ball, all the same 42 75 169 40 76
size. If a ball is chosen from the box at a Going on his past performances, what is
random, what is the chance of getting: the probability that Liam’s next score
a a green ball? will be:
b a yellow ball? i greater than 80? ii less than 70?
c a green or yellow ball? b Liam did not score over 170. Does this
d a blue ball? mean that the probability of him
scoring 170 or more is zero? Comment
4 A game is played by picking counters on your answer.
numbered 1 to 50 out of a bag. The table
shows the results for picking various 6 When two teams play football there are
counters. three possible results. Each team could win
and the game could be drawn. Therefore
Counter Result the probability that a particular team wins
1 Win $1 is 1--3- . Is this statement correct? Justify your
answer.
A prime number Win 50c
A number ending No result
with a 4 or a 6
Any other number Lose 50c

1 Rolling two dice


Probability
2 Pack of cards

CHAPTER 14 PROBABILITY 401


Page 232 of 525
IM3_Ch14.fm Page 402 Friday, January 4, 2008 9:29 AM

men
gn t
Chapter 14 | Working Mathematically
assi

1 Use ID Card 6 on page xviii to identify: 3 Through how many degrees does the spoke
14B a 6 b 7 c 8 d 12 of a wheel turn in 5 minutes if the wheel is
e 14 f 15 g 17 h 18 turning 700 revolutions per minute?
i 19 j 22
4 Indu is eight years younger than John, but
2 Use the Algebra Card on page xxii to: John is twice her age. How old is Indu?
a add column M to column N
b subtract column M from column N
c add columns N and O
d find the value of the terms in J if x = −2

5 This graph appeared in 1992. The scale has been removed. 100% of sales are represented here.
Disposable nappies
% Brand Shares

Supermarket sales
$165 million

Huggies Pampers Snugglers VIP Housebrand/


Generics/others

Estimate the percentage of the sales belonging to Huggies if:


a the height of each picture is the significant measure
b the area of each picture is the significant measure
c the volume of the child represented in each picture is the significant measure

402 INTERNATIONAL MATHEMATICS 3


Page 233 of 525
IM3_Ch14.fm Page 403 Friday, January 4, 2008 9:29 AM

6 The graph is a comparison of the stopping distances of a car and truck at speeds of 60 km/h,
80 km/h and 100 km/h.
Comparison of stopping distances
Car The first section of each column
60 km/h represents the distance travelled in
Truck the one second it takes an average
60 km/h driver to react. The second section
is the average braking distance
Car under good weather, road and
80 km/h vehicle conditions.
Truck
80 km/h
Car
100 km/h
Truck
100 km/h
0 10 20 30 40 50 60 70 80 90 100 110 120 130 140
Distance in metres

The reaction distance is the distance travelled by the vehicle in the time it takes the driver to
take his/her foot off the accelerator and apply the brakes. The braking distance is the distance
travelled after the brakes are applied. Stopping distance = reaction distance + braking distance.
a What is the reaction distance at:
i 60 km/h?
ii 80 km/h?
iii 100 km/h?
b What is the braking distance for:
i a truck travelling at 80 km/h?
ii a car travelling at 80 km/h?
c What is the difference in stopping distances
for a car and truck travelling at:
i 60 km/h?
ii 80 km/h?
iii 100 km/h?
d If a truck is following a car, both
travelling at 80 km/h and both drivers
brake simultaneously, find:
i the braking distance of the car
ii the braking distance of the truck
iii how far the truck must be behind the car to avoid a smash

CHAPTER 14 PROBABILITY 403


Page 234 of 525
Probability
Chapter
13
Contents: A Probability by experiment
B Chance investigations
C Sample space
D Theoretical probability
E Using grids to find probabilities
F Multiplying probabilities
G Using tree diagrams
H Sampling with and without
replacement
I Probabilities from Venn diagrams
(Extension)
J Conditional probability (Extension)

Page 235 of 525


100

100

100

100
50

50

50

50
0

0
25

75

95

25

75

95

25

75

95

25

75

95
5

IB_PD
cyan magenta yellow black

Y:\HAESE\IB_PD\IBPD_13\305ibpd13.cdr Wednesday, 5 July 2006 3:36:07 PM PETERDELL


306 PROBABILITY (Chapter 13)

Often we are confronted by situations where we are not sure what outcome will occur.
We hear statements such as:
“The Wildcats will probably beat the Tigers on Saturday.”
“It is unlikely that it will rain today.”
“I probably will make the team.”
Each of these statements indicates a likelihood (or chance) of a particular event happening.
We can indicate the likelihood of an event happening in the future by using a percentage.

0% indicates we believe the event will not occur.


100% indicates we believe the event is certain to occur.

All events can therefore be assigned a percentage between 0% and 100% (inclusive).
A number close to 0% indicates the event is unlikely to occur, whereas a number close to
100% means that it is highly likely to occur.
In mathematics, we usually use either decimals or fractions rather than percentages for prob-
abilities. However, as 100% = 1, comparisons or conversions from percentages to fractions
or decimals are very simple.

An impossible event which has 0% chance of happening is assigned


a probability of 0.
A certain event which has 100% chance of happening is assigned a
probability of 1.
All other events can then be assigned a probability between 0 and 1.

The assigning of probabilities is usually based on either


² observing the results of an experiment (experimental probability), or
² using arguments of symmetry (theoretical probability).

HISTORICAL NOTE
² Jerome Cardan (1501 to 1576) admitted in his autobiography that he
gambled “not only every year, but .... every day, and with the loss at once
of thought, of substance, and of time”. He wrote a handbook on gambling
with tips on cheating and how to detect it. His book included discussion
on equally likely events, frequency tables for dice probabilities, and
expectations.

² Laplace (1749 - 1827) once described the theory of probabil-


ity as “nothing but common sense reduced to calculation”.

² Blaise Pascal (1623 - 1662) invented the first mechanical


digital calculator. Pascal and his friend Fermat were first to
develop probability theory as we know it today. Pascal also
developed the syringe and the hydraulic press. He wrote a
large number of articles on Christian beliefs and ethics. Blaise Pascal

Page 236 of 525


100

100

100

100
50

50

50

50
0

0
25

75

95

25

75

95

25

75

95

25

75

95
5

IB_PD
cyan magenta yellow black

Y:\HAESE\IB_PD\IBPD_13\306ibpd13.cdr Wednesday, 5 July 2006 3:37:01 PM PETERDELL


PROBABILITY (Chapter 13) 307

OPENING PROBLEM
When Karla dropped some metal nuts she
0
noticed that they landed either on their ends
or on their sides. She then tossed a nut 200
times and it landed on its end 137 times.
Later Sam repeated the experiment and the
nut finished on its end 145 times.
side end
For you to consider:
² What would Karla’s best estimate be of the chance that the nut will finish on its end?
² What would Sam’s estimate be?
² How can a better estimate of the chance of an end occurring be made?
² Hilda said that the best estimate would be obtained when the nut is tossed thousands
of times. Is she correct?

A PROBABILITY BY EXPERIMENT
In an experiment we should use suitable language to help us describe what we are doing and
the results we expect and get.

² The number of trials is the total number of times the experiment is repeated.
² The outcomes are the different results possible for one trial of the experiment.
² The frequency of a particular outcome is the number of times this outcome is observed.
² The relative frequency of an outcome is the frequency of that outcome divided by the
total number of trials.

So, if we flip a tin can into the air 250 times and if it
lands on an end 37 times, then
² the number of trials is 250
² the outcomes are ends and sides
² the frequency of ends is 37 and sides is 213
² the relative frequency of ends is 0:148 and sides
is 0:852:

EXPERIMENTAL PROBABILITY
Sometimes the only way of finding the chance of occurrence of a particular event is by
experiment.
Tin can flipping is one such example. The chance of a can of this shape finishing on its end
is the relative frequency found by experimentation.
We say, the estimated experimental probability is the relative frequency of the event.
We write: Experimental P(end) = 0:148
Page 237 of 525
100

100

100

100
50

50

50

50
0

0
25

75

95

25

75

95

25

75

95

25

75

95
5

IB_PD
cyan magenta yellow black

Y:\HAESE\IB_PD\IBPD_13\307ibpd13.cdr Wednesday, 5 July 2006 3:37:11 PM PETERDELL


308 PROBABILITY (Chapter 13)

Note: ² The larger the number of trials, the more confident we are in the experimental
probability obtained.
² Experimental P(::::::) = relative frequency of ......

Example 1 Self Tutor


A marketing company surveys 50 ran- Brand Frequency Rel. Freq.
domly selected people to discover what
Shine 10 0:20
brand of toothpaste they use. The results
Starbright 14 0:28
are tabulated.
Brite 4 0:08
a Based on these results, what would be
the experimental probability of a com- Clean 12 0:24
munity member using: No Name 10 0:20
i Starbright ii Clean?
b Would you classify the estimate of a to be very good, good, or poor? Why?

a i Expt P(Starbright) = 0:28 ii Expt P(Clean) = 0:24


b Poor, as the sample size is very small.

EXERCISE 13A
1 A marketing company was commissioned to investigate brands of products usually found
in the bathroom. The results of a soap survey are below:
a How many people were randomly selected in
Brand Count Relative
this survey? Frequency
b Calculate the relative frequency of use of each Silktouch 115
brand of soap.
Super 87
c Using these results, what is the experimental Just Soap 108
probability that the soap used by a randomly
Indulgence 188
selected person is:
Total
i Just Soap ii Indulgence iii Silktouch?

2 Two coins were tossed 356 times and the number of heads occurring at each toss was
recorded. The results were:
a Copy and complete the table given.
Outcome Freq. Rel. Freq.
b What is the best estimate of the chance of
0 heads 89
the following events occurring from this
1 head
data?
2 heads 95
Total i 0 heads ii 1 head iii 2 heads

3 At the Annual Show the fairy floss vendor estimated that three times as many people
preferred pink fairy floss to white fairy floss.
a If 250 people wanted white fairy floss, estimate how many wanted pink.
b What is the estimate of the probability that the next customer asks for:
i white fairy floss ii pink fairy floss?
Page 238 of 525
100

100

100

100
50

50

50

50
0

0
25

75

95

25

75

95

25

75

95

25

75

95
5

IB_PD
cyan magenta yellow black

Y:\HAESE\IB_PD\IBPD_13\308ibpd13.cdr Wednesday, 5 July 2006 3:37:46 PM PETERDELL


PROBABILITY (Chapter 13) 309

B CHANCE INVESTIGATIONS
In experimental probability, the relative frequency of an outcome gives us an estimate for
the probability of that outcome.
In general, the greater the number of trials, the more we can rely on our estimate of the
probability.
The most commonly used equipment for experimental probability and games of chance is
described below:

COINS
When a coin is tossed there are two possible sides that could show upwards: the head (usually
the head of a monarch, president or leader) and the tail (the other side of the coin). We would
expect a head (H) and a tail (T) to have equal chance of occurring, i.e., we expect each to
occur 50% of the time. So,
the probability of obtaining a head is 12 , and

the probability of obtaining a tail is 12 :

The table alongside shows actual Number of tosses H T %H %T


experimental results obtained for
10 7 3 70:0 30:0
tossing a coin:
100 56 44 56:0 44:0
1000 491 509 49:1 50:9

each is nearly 50%


These experimental results support our expectations and suggest the general rule:
“The more times we repeat an experiment, like tossing a coin, the closer the results will be
to the theoretical results we expect to obtain”.

DICE (Note: Dice is the plural of die.)


The most commonly used dice are small cubes with the
numbers 1, 2, 3, 4, 5 and 6 marked on them using dots.
The numbers on the face of a cubic die are arranged such
that the sum of each pair of opposite faces is seven.

SPINNERS
A simple spinner consists of a regular polygon (or sometimes a circle
with equal sectors) with a toothpick or match through its centre.
To the right is a spinner showing a result of 1
since it has come to rest on the side marked 1.

A spinner such as the one on the left may be used instead of a die,
providing all angles are equal. The result shown is 2, since the pointer
came to rest on the sector marked 2.
Page 239 of 525
100

100

100

100
50

50

50

50
0

0
25

75

95

25

75

95

25

75

95

25

75

95
5

IB_PD
cyan magenta yellow black

Y:\HAESE\IB_PD\IBPD_13\309ibpd13.cdr Wednesday, 5 July 2006 3:37:58 PM PETERDELL


310 PROBABILITY (Chapter 13)

INVESTIGATION 1 ROLLING A DIE


You will need:
At least one normal six-sided die with numbers 1 to 6 on its faces.
Several dice would be useful to speed up the experiment.
What to do:
1 Examine a die. List the possible outcomes for the uppermost face
when the die is rolled.
2 Consider the possible outcomes when the die is rolled 60 times. Complete a table of
your expected results which includes columns of Outcomes, Expected relative
frequency and Expected frequency.
Outcome Tally Frequency Rel. frequ.
3 Roll the die 60 times and record
the result on the uppermost face in 1
a table like the one alongside: 2
..
4 Pool as much data as you can with .
other students. 6
² Look at similarities and differ- Total 60
ences from one set to another.
² Look at the overall pooled data added into one table. SIMULATION

5 How close to your expectation were your results?


6 Use the die rolling simulation from the computer package on the CD to
roll the die 10 000 times and repeat this 10 times. On each occasion, record your
results in a table like that in 3. Do your results confirm your expected results?

INVESTIGATION 2 TOSSING THREE COINS


You will need:

Three coins. They do not


have to be all the same type.
What to do:
1 List the 8 possible outcomes for tossing 3 coins simultaneously. (Let HHT represent
“a head for the first coin, a head for the second and a tail for the third”.)
2 How many of these outcomes could be described as:
a ‘three heads’ b ‘two heads and a tail’
c ‘one head and two tails’ d ‘three tails’?
3 Consider the possible outcomes of tossing three coins 80 times. Copy and complete
a table of expected results.
Result Expected relative frequency Expected frequency
3 heads
2 heads and 1 tail
1 head and 2 tails
3 tails

Page 240 of 525


100

100

100

100
50

50

50

50
0

0
25

75

95

25

75

95

25

75

95

25

75

95
5

IB_PD
cyan magenta yellow black

Y:\HAESE\IB_PD\IBPD_13\310ibpd13.cdr Wednesday, 5 July 2006 3:38:12 PM PETERDELL


PROBABILITY (Chapter 13) 311

4 Now perform the above experiment and record your results in a table such as that
which follows:

Result Tally Frequency Relative frequency


3 heads
2 heads and 1 tail
1 head and 2 tails
3 tails
Total 80 1

5 Pool as much data as you can with other students and find the overall relative fre-
quencies of the results.
SIMULATION
² Look for similarities and differences from one set to another.
² Look at the overall pooled data added into one table.
6 Comment on any differences between the observed results and the expected results
for your own experiment and the results of the group overall.
7 Use the three coin simulation from the computer package on the CD to toss the coins
5000 times. Repeat this 10 times and on each occasion record your results in a table
like that in 4. Are your results consistent with your expectations?

INVESTIGATION 3 ROLLING A PAIR OF DICE


You will need: 2 normal six-sided dice with numbers 1 to 6 on the faces.
What to do:
1

The illustration above shows that when two dice are rolled there are 36 possible out-
comes. Of these, f1, 3g, f2, 2g and f3, 1g give a sum of 4.
Using the illustra-
tion above, copy and Sum 2 3 4 5 ¢¢¢ 12
complete the follow- Fraction of total 3
36
ing table of expected
Fraction as decimal 0:083
(theoretical) results:
2 If a pair of dice is rolled 360 times, how many of each result (2, 3, 4, ...., 12) would
you expect to get? Extend your table of 1 by adding another row and write your
expected frequencies within it.
Page 241 of 525
100

100

100

100
50

50

50

50
0

0
25

75

95

25

75

95

25

75

95

25

75

95
5

IB_PD
cyan magenta yellow black

Y:\HAESE\IB_PD\IBPD_13\311ibpd13.cdr Wednesday, 5 July 2006 3:39:00 PM PETERDELL


312 PROBABILITY (Chapter 13)

3 Toss the two dice 360 times and record in a table the sum of the two numbers for each
toss.
Sum Tally Frequency Relative Frequency
2
SIMULATION
3
4
5
PRINTABLE ..
WORKSHEET .
12
Total 360 1

4 Use the two dice simulation from the computer package on the CD to roll the pair of
dice 10 000 times. Repeat this 10 times and on each occasion record your results in
a table like that of 3. Are your results consistent with your expectations?

C SAMPLE SPACE
A sample space is the set of all possible outcomes of an experiment.

There are a variety of ways of representing or illustrating sample spaces.

LISTING OUTCOMES
Example 2 Self Tutor
List the sample space for: a tossing a coin b rolling a die.

a When a coin is tossed, there b When a die is rolled, there are 6


are two possible outcomes. possible outcomes.
) sample space = fH, Tg ) sample space = f1, 2, 3, 4, 5, 6g

2-DIMENSIONAL GRIDS
When an experiment involves more than one operation we can still use listing to illustrate the
sample space. However, a grid can often be a better way of achieving this.

Example 3 Self Tutor


Illustrate the possible outcomes when 2 coins are tossed by using a
2-dimensional grid.

coin 2 Each of the points on the grid represents


T one of the possible outcomes:
fHH, HT, TH, TTg
H
coin 1
H T
Page 242 of 525
100

100

100

100
50

50

50

50
0

0
25

75

95

25

75

95

25

75

95

25

75

95
5

IB_PD
cyan magenta yellow black

Y:\HAESE\IB_PD\IBPD_13\312ibpd13.cdr Wednesday, 5 July 2006 3:39:23 PM PETERDELL


PROBABILITY (Chapter 13) 313

TREE DIAGRAMS
The sample space in Example 3 could also be represented by a tree diagram.
coin 1 coin 2
H HH
H Each “branch” shows a different
T HT
H TH
outcome. Once again we see the
T sample space is fHH, HT, TH, TTg.
T TT

The advantage of tree diagrams is that they can be used when more than two operations are
involved.

Example 4 Self Tutor


Illustrate, using a tree diagram, the possible marble 1 marble 2
outcomes when drawing two marbles from R
a bag containing a number of red, green and R G
yellow marbles. Y
R
G G
Y
R
Y G
Y

EXERCISE 13C
1 List the sample space for the following:
a rolling a six-sided die
b the sexes of a 3-child family
c the order in which 3 boys can be lined up
d the order in which 4 different rowing teams could finish a race.

2 Illustrate on a 2-dimensional grid the sample space for:


a rolling a die and tossing a coin simultaneously
b rolling two dice
c twirling a square spinner labelled A, B, C, D and
a pentagonal spinner labelled 1, 2, 3, 4, 5
d tossing two coins and rolling a die simultaneously.

3 Illustrate on a tree diagram the sample space for:


a tossing a 20-cent and 50-cent coin simultaneously
b tossing 3 different coins
c the sexes of a 4-child family
d two teams, X and Y, play soccer. The team which is first to kick 3 goals wins the
match.

Page 243 of 525


100

100

100

100
50

50

50

50
0

0
25

75

95

25

75

95

25

75

95

25

75

95
5

IB_PD
cyan magenta yellow black

Y:\HAESE\IB_PD\IBPD_13\313ibpd13.cdr Wednesday, 5 July 2006 3:39:58 PM PETERDELL


314 PROBABILITY (Chapter 13)

D THEORETICAL PROBABILITY
Consider the pentagonal spinner alongside.
Since the spinner is symmetrical, when it is spun the
arrowed marker could finish with equal likelihood on
each of the sections marked 1 to 5:
Therefore, we would say that the likelihood of obtaining
a particular number, for example, 4, would be
1 chance in 5, 5,
1
20% or 0:2

This is a mathematical (or theoretical) probability and is based on what we theoretically


expect to occur.

The theoretical probability of a particular event is the chance of that event


occurring in any trial of the experiment.

Consider the event of getting a result of 3 or more from one spin of the pentagonal spinner.
There are three favourable outcomes (3, 4 or 5) out of the five possible outcomes, and each
of these is equally likely to occur.
We read Et_ as
So, P(3 or more) = 3
5
‘3 chances in 5’.

In general, for an event E containing equally likely possible results:

the number of outcomes of the event E


P(E) = .
the total number of possible outcomes

Example 5 Self Tutor


A marble is randomly selected from a bag containing 1 green, 4 yellow and
4 red marbles. Determine the probability of getting:
a a red marble b a green or yellow marble
c a blue marble d a green, yellow or red marble

The sample space is fG, Y, Y, Y, Y, R, R, R, Rg


which has 1 + 4 + 4 = 9 outcomes.
1+4
a P(red) b P(a green or a yellow) =
9
4
= 9 = 5
9

1+4+4
c P(blue) d P(green, yellow or red) =
9
0
= 9 = 9
9
=0 =1
Page 244 of 525
100

100

100

100
50

50

50

50
0

0
25

75

95

25

75

95

25

75

95

25

75

95
5

IB_PD
cyan magenta yellow black

Y:\HAESE\IB_PD\IBPD_13\314ibpd13.cdr Wednesday, 5 July 2006 3:40:21 PM PETERDELL


PROBABILITY (Chapter 13) 315

In Example 5 notice that in c a blue result cannot occur and the calculated probability is 0,
which fits the fact that it has no chance of occurring.
Also notice in d, a green, yellow or red result is certain to occur. It is 100% likely which is
perfectly described using a 1.
The two events of no chance of occurring with probability 0 and
certain to occur with probability 1 are two extremes.

Consequently, for any event E, 0 6 P(E) 6 1:

COMPLEMENTARY EVENTS
Example 6 Self Tutor
An ordinary 6-sided die is rolled once. Determine the chance of:
a getting a 5 b not getting a 5
c getting an even number d getting an odd number

The sample space of possible outcomes is f1, 2, 3, 4, 5, 6g


a P(5) b P(not getting a 5)
= 1
6
= P(1, 2, 3, 4 or 6)
= 56

c P(an even number) d P(an odd number)


= P(2, 4 or 6) = P(1, 3 or 5)
= 36 = 36
1 1
= 2 = 2

In Example 6, did you notice that


P(5) + P(not getting a 5) = 1 and that P(an even number) + P(an odd number) = 1?
This is no surprise as getting a 5 and not getting a 5 are complementary events where one
of them must occur.
Likewise getting an even number and getting an odd number are complementary events.

NOTATION
If E is an event, then E 0 is the complementary event of E:

So, P(E) + P(E 0 ) = 1.

A useful rearrangement is: P(E not occurring) = 1 ¡ P(E occurring)

EXERCISE 13D
1 A fair die is rolled. Determine the probability of getting:
a a 2 or a 3 b a positive integer c a7
d a result greater than 4 e a non-six
Page 245 of 525
100

100

100

100
50

50

50

50
0

0
25

75

95

25

75

95

25

75

95

25

75

95
5

IB_PD
cyan magenta yellow black

Y:\HAESE\IB_PD\IBPD_13\315ibpd13.cdr Wednesday, 5 July 2006 3:40:38 PM PETERDELL


316 PROBABILITY (Chapter 13)

2 A symmetrical octahedral die has numbers 1 to 8 marked on its faces, and it is rolled
once. Determine the probability of throwing:
a a2 b a number less than 4
c a number less than 1 d a number between 1 and 8

3 A regular pentagonal spinner has the numbers


1 to 5 marked on its partitions. Determine the
probability that after a spin the result will be:
a even
b prime
c a factor of 6.

4 A bag contains 3 red and 7 blue buttons, and a button is randomly selected from the bag.
Determine the probability of getting:
a a red b a blue c a red or a blue d a red and a blue.

5 One ticket is chosen in a lottery consisting of 100


tickets numbered 1 to 100 and the choice is made
randomly. Determine the probability that the ticket
is:
a a two digit number b a multiple of 12
c a multiple of 7 or 11

6 Determine the probability that a person randomly


selected in the street has his (or her) birthday in
September.
7 List the six different orders in which buses A, B and C can arrive at a bus stop. If the
three buses arrive at random at a bus stop, determine the probability that:
a bus A arrives first b they arrive in alphabetical order
c bus B is the second bus to arrive d after bus B, bus C will be next to arrive

8 a List the 8 possible 3-child families, according to the gender of the children. For
example, GGB means “the first is a girl, the second is a girl, and the third is a
boy”.
b Assuming that each of these is equally likely to occur, determine the probability
that a randomly selected 3-child family consists of:
i all boys ii all girls
iii boy, then boy, then girl iv two girls and a boy
v a boy for the eldest vi at least one girl

9 a List, in systematic order, the 24 different orders in which four blocks W, X, Y and
Z can be placed one on top of the other.
b Hence, determine the probability that when 4 blocks are placed at random on top
of one another:
i W is at the bottom ii X is in one of the two middle positions
iii X and Y are placed together
iv X, Y and Z are together, not necessarily in that order.
Page 246 of 525
100

100

100

100
50

50

50

50
0

0
25

75

95

25

75

95

25

75

95

25

75

95
5

IB_PD
cyan magenta yellow black

Y:\HAESE\IB_PD\IBPD_13\316ibpd13.cdr Wednesday, 5 July 2006 3:40:48 PM PETERDELL


PROBABILITY (Chapter 13) 317

E USING GRIDS TO FIND PROBABILITIES


Two-dimensional grids give us excellent visual displays of sample spaces. From these we
can count favourable outcomes and so calculate probabilities.
coin B This point represents ‘a tail from coin A’ and ‘a tail from coin B’.
T This point represents ‘a tail from coin A’ and ‘a head from coin B’.
H
There are four members of the sample space.
H T coin A

Example 7 Self Tutor


Use a two-dimensional grid to illustrate the sample space for tossing a coin and
rolling a die simultaneously. From this grid determine the probability of:
a tossing a tail b rolling a 3
c getting a head and a 6 d getting a head or a 6

There are 12 members in the sample space.


coin a P(tail) = 126
= 12
T
b P(3) = 2
12 = 1
6
H

1 2 3 4 5 6
die c P(head and a ‘6’) = 1
12

d P(head or a ‘6’) = 7
12 fthe enclosed pointsg

Example 8 Self Tutor


Two circular spinners, each with 1, 2, 3 and 4 on their sectors, are twirled
simultaneously. Draw a two-dimensional grid of the possible outcomes.
Use your grid to determine the probability of getting:
a a 2 with each spinner b a 2 and a 4
c an even result for each spinner

The sample space has 16 members.


spinner 2 a P(a 2 with each spinner) = 1
16

4 b P(a 2 and a 4) = 2
fcrossed pointsg
16
3 1
= 8
2
1 c P(an even result for each spinner)
spinner 1 4
= 16 fcircled pointsg
1 2 3 4
1
= 4

Page 247 of 525


100

100

100

100
50

50

50

50
0

0
25

75

95

25

75

95

25

75

95

25

75

95
5

IB_PD
cyan magenta yellow black

Y:\HAESE\IB_PD\IBPD_13\317ibpd13.cdr Wednesday, 5 July 2006 3:41:06 PM PETERDELL


318 PROBABILITY (Chapter 13)

EXERCISE 13E
1 Two coins are tossed simultaneously. Use a 2-dimensional grid to illustrate the sample
space and hence determine the probability of getting:
a two heads b a head and a tail c at least one tail.

2 Draw a 2-dimensional grid to illustrate the sample space when an ordinary die (numbered
1 to 6) is tossed and a square spinner labelled A, B, C and D is spun. Hence, determine
the probability of getting:
a A and 4 b B and a prime number c a consonant and a multiple of 3.

3 A pair of dice is rolled. The 36 different possible ‘pair of dice’ results are illustrated
below on a 2-dimensional grid.
Use the 2-dimensional grid of the 36 possible outcomes to determine the probability of
getting:
a two 6’s
b a 2 and a 3 die 2
c a 2 or a 3 6
d at least one 4 5
4
e exactly one 4
3
f no fives
2
g two even numbers 1
h a sum of 8 die 1
1 2 3 4 5 6
i a sum greater than 6
j a sum of no more than 6.

F MULTIPLYING PROBABILITIES
In the previous section we used two-dimensional grids to represent sample spaces and hence
find answers to certain types of probability problems.
Consider again a simple example of tossing coin
a coin and rolling a die simultaneously.
T
H
die
1 2 3 4 5 6

When asked “What is the probability of getting a head and a 5?” we get an answer of 12 1

from the grid since there are 12 possible outcomes but only one with the property that we
want.
But P(head) = 12 and P(a ‘5’) = 16 and 12 £ 16 = 12 1

This suggests that P(head and a ‘5’) = P(head) £ P(‘5’),


i.e., we multiply the separate probabilities.
In general:
If A and B are two events then P(A and B) = P(A) £ P(B).
Page 248 of 525
100

100

100

100
50

50

50

50
0

0
25

75

95

25

75

95

25

75

95

25

75

95
5

IB_PD
cyan magenta yellow black

Y:\HAESE\IB_PD\IBPD_13\318ibpd13.cdr Wednesday, 5 July 2006 3:41:26 PM PETERDELL


PROBABILITY (Chapter 13) 319

Example 9 Self Tutor


Joe has probability 34 of hitting a target and Anne has probability 45 .
If they both fire simultaneously at the target, determine the probability that:
a they both hit it b they both miss it.

a P(both hit) b P(both miss)


= P(Joe and Anne hit) = P(Joe misses and Anne misses)
= P(Joe hits) £ P(Anne hits) = P(Joe misses) £ P(Anne misses)
3 4 1 1
= 4 £ 5 = 4 £ 5
3 1
= 5 = 20

EXERCISE 13F
1 Janice and Lee take set shots at a
netball goal from 3 m. From past
experience Janice throws a goal 2
times in every 3 shots on average
whereas Lee throws a goal 4 times in
every 7. If they both shoot for goals
determine the probability that:

a both score a goal


b both miss
c Janice throws a goal but Lee misses.

2 When a nut was tossed 500 times it finished on its edge 156 times
and on its side for the rest. Use this information to estimate the
probability that when two identical nuts are tossed:
a they both fall on their edges b they both fall on their sides.

3 Helena has probability 13 of hitting a target with an


arrow, whilst Marco has probability 25 .
If they fire at the target, determine the probability that:
a both hit the target
b both miss the target
c Helena hits the target and Marco misses it
d Helena misses the target and Marco hits it.

4 A certain brand of drawing pin was tossed into the air 1000 times and it landed on its
back 375 times and on its side for the remainder. Use this information to
estimate the probability that:
a one drawing pin, when tossed, will fall on its i back ii side
b two drawing pins, when tossed, will both fall on their backs
c two drawing pins, when tossed, will both fall on their sides.
Page 249 of 525
100

100

100

100
50

50

50

50
0

0
25

75

95

25

75

95

25

75

95

25

75

95
5

IB_PD
cyan magenta yellow black

Y:\HAESE\IB_PD\IBPD_13\319ibpd13.cdr Wednesday, 5 July 2006 3:41:42 PM PETERDELL


320 PROBABILITY (Chapter 13)

G USING TREE DIAGRAMS


Tree diagrams can be used to illustrate sample spaces provided that the alternatives are not
too numerous.
Once the sample space is illustrated, the tree diagram can be used for determining probabilities.
Consider Example 9 again. The tree diagram for this information is:

outcome probability
H = hit Anne’s results
M = miss Joe’s results tR_ H H and H Er_ £ tR_ = Qw_Wp_
H
Er_ M H and M Er_ £ tQ_ = Dw_p_
tQ_
tR_ H M and H rQ_ £ tR_ = Fw_p_
rQ_
M
tQ_ M M and M rQ_ £ tQ_ = Aw_p_
total 1
Notice that:
² The probabilities for hitting and missing are marked on the branches.
² There are four alternative paths and each branch shows a particular outcome.
² All outcomes are represented and the probabilities are obtained by multiplying.

Example 10 Self Tutor


Sylke has bad luck with the weather when she takes her summer holidays.
She estimates that it rains 60% of the time and it is cold 70% of the time.
a Draw a tree diagram to illustrate this situation.
b Use the tree diagram to determine the chance that for Sylke’s holidays:
i it is cold and raining ii it is fine and cold.

a C = the weather is cold R = it is raining


outcome probability
temperature rain
0.6 R C and R 0.7 £ 0.6 = 0.42
C
0.7 R' C and R' 0.7 £ 0.4 = 0.28
0.4
0.6 R C' and R 0.3 £ 0.6 = 0.18
0.3
C'
0.4 R' C' and R' 0.3 £ 0.4 = 0.12
total 1.00

b i P(it is cold and raining) ii P(it is fine and cold)


= P(C and R) = P(R0 and C)
= 0:7 £ 0:6 = 0:4 £ 0:7
= 0:42 = 0:28
Page 250 of 525
100

100

100

100
50

50

50

50
0

0
25

75

95

25

75

95

25

75

95

25

75

95
5

IB_PD
cyan magenta yellow black

Y:\HAESE\IB_PD\IBPD_13\320ibpd13.cdr Wednesday, 5 July 2006 3:42:09 PM PETERDELL


PROBABILITY (Chapter 13) 321

EXERCISE 13G
1 Jar A contains 2 white and 3 red marbles, whereas Jar B jar marble
contains 4 red and 1 white marble. A jar is randomly selected R
and one marble is taken from it. A
W
a Copy and complete the branches on the tree diagram shown.
b Determine the probability that the marble is red.

2 A square dartboard is made up of nine smaller squares.


Five are painted red and the remainder are black.
R is “landing on red” B is “landing on black”
a Evaluate: i P(R) ii P(B)
b Copy and complete the tree diagram illustrating two
darts being thrown at the board.
c Using b, determine the probability that:
dart 1 dart 2
i both darts land on black
ii both darts land on red
iii dart 1 lands on red and dart 2 lands on black
iv one dart lands on red and the other lands on black.
3 The probability of rain tomorrow is estimated to be 15 . If it does rain, the Mudlarks
football team will have an 80% probability of winning. If it is fine they have a 50%
chance of winning. Display the sample space of possible results of the football match
on a tree diagram.
Determine the probability that the Mudlarks will win tomorrow.
4 A factory produces bottles of fruit juice. Machine A fills 60% of the bottles produced
and machine B fills the rest. Machine A underfills 1% of the bottles, while Machine B
underfills 0:5%. Determine the probability that the next bottle inspected at this factory
is underfilled.

Example 11 Self Tutor


Bag A contains 3 red and 2 yellow tickets. Bag B contains 1 red and 4 yellow
tickets. A bag is randomly selected by tossing a coin and one ticket is removed from
it. Determine the probability that it is yellow.
To get a yellow ticket
we take either the first
ticket outcome branch marked with a
Bag A Bag B bag 3–
X or the second one
5 R A and R
–1 A marked with a X and
2 Y A and Y X
2– add the probabilities.
3R 1R 5
1–
2Y 4Y –1
5 R B and R
2 B
4– Y B and Y X
5

P(yellow) = P(A and Y ) + P(B and Y )


= 12 £ 25 + 12 £ 45 fbranches marked with a Xg
= 6
10 (or 35 )
Page 251 of 525
100

100

100

100
50

50

50

50
0

0
25

75

95

25

75

95

25

75

95

25

75

95
5

IB_PD
cyan magenta yellow black

Y:\HAESE\IB_PD\IBPD_13\321ibpd13.cdr Wednesday, 5 July 2006 3:42:29 PM PETERDELL


322 PROBABILITY (Chapter 13)

5 Three bags contain different numbers of blue and red


2B 1B 2B
marbles. A bag is selected at random and one marble
is then selected from it.
2R 3R 2R
A B C Determine the probability that it is: a blue b red.

H SAMPLING WITH AND


WITHOUT REPLACEMENT
Consider a box containing 3 red, 2 blue and 1 yellow marble.
Suppose we wish to sample two marbles: R
² by replacement of the first before the second is drawn Y R
² by not replacing the first before the second is drawn. R B B
Examine how the tree diagrams differ:
With replacement Without replacement
2nd 2nd
1st Ey_ R 1st Wt_ R
R Wy_ B ( ) R Wt_ B ( )
Ey_
Qy_ Y ( ) Ey_
Qt_ Y ( )
Ey_ R ( ) Et_ R ( )
Wy_ B Wy_ B Wy_ B Qt_ B
Qy_ Y ( ) Qt_ Y ( )
Qy_ R ( ) Qy_ Et_ R ( )
Ey_
Y Wy_ B ( ) Y Wt_ B ( )
Qy_ Y can’t
have
YY
This branch represents Blue with the 1st draw and Red with the second, written as BR.
Notice that: ² with replacement P(two reds) = 3
6 £ 3
6 = 1
4

² without replacement P(two reds) = 3


6 £ 2
5 = 1
5

Example 12 Self Tutor


For the example of the box containing 3 red, 2 blue and 1 yellow marble find the
probability of getting two different colours:
a if replacement occurs b if replacement does not occur.

a P(two different colours)


= P(RB or RY or BR or BY or Y R or Y B) fticked onesg
3 2 3 1 2 3 2 1 1 3 1 2
= 6 £ 6 + 6 £ 6 + 6 £ 6 + 6 £ 6 + 6 £ 6 + 6 £ 6
11
= 18

Page 252 of 525


100

100

100

100
50

50

50

50
0

0
25

75

95

25

75

95

25

75

95

25

75

95
5

IB_PD
cyan magenta yellow black

Y:\HAESE\IB_PD\IBPD_13\322ibpd13.cdr Wednesday, 5 July 2006 3:42:45 PM PETERDELL


PROBABILITY (Chapter 13) 323

b P(two different colours)


= P(RB or RY or BR or BY or Y R or Y B) fcrossed onesg
3 2 3 1 2 3 2 1 1 3 1 2
= 6 £ 5 + 6 £ 5 + 6 £ 5 + 6 £ 5 + 6 £ 5 + 6 £ 5
11
= 15

Note:
When using tree diagrams to assist in solving probability questions, the following rules should
be used:
² The probability for each branch is calculated by multiplying the probabilities along
that path.
² If two or more branch paths meet the description of the compound event, the
probability of each path is found and then they are added.

EXERCISE 13H
1 A box contains 5 red and 2 white tickets. Two tickets are drawn at random (the first
being replaced before the second is drawn). Draw a tree diagram to represent the sample
space and use it to determine the probability that:
a both are red b both are white
c the first is red and the second is white d one is red and the other is white
2 7 tickets numbered 1, 2, 3, 4, 5, 6 and 7 are placed in a hat. Two of the tickets are taken
from the hat at random without replacement. Determine the probability that:
a both are even b both are odd
c the first is odd and the second is even d one is odd and the other is even
3 Amelie has a bag containing two different varieties of apples. They are approximately
the same size and shape, but one variety is red and the other is green. There are 4 red
apples and 6 green ones. She selects one apple at random, eats it and then takes another,
also at random. Determine the probability that:
a both apples were red
b both apples were green
c the first was red and the second was green
d the first was green and the second was red
Add your answers to a, b, c and d. Explain why the answer must be 1.

4 Marjut has a carton containing 10 cans of soup. 4 cans are tomato and the rest are
pumpkin. She selects 2 cans at random without looking at the labels.
a Let T represent tomato and P represent pumpkin. Draw a tree
diagram to illustrate this sampling process.
b What is the probability that both cans were tomato soup?
c What is the probability that one can was tomato and the
other can was pumpkin soup?

Page 253 of 525


100

100

100

100
50

50

50

50
0

0
25

75

95

25

75

95

25

75

95

25

75

95
5

IB_PD
cyan magenta yellow black

Y:\HAESE\IB_PD\IBPD_13\323ibpd13.cdr Tuesday, 11 July 2006 12:38:35 PM PETERDELL


324 PROBABILITY (Chapter 13)

5 Box A contains 2 red and 2 green tickets. Box B contains 3 red and 5 green tickets.
When a box has been selected at random, two tickets are randomly selected without
replacement from it. Determine the probability that:
a both are green b they are different in colour.

6 A bag contains two white and five red marbles. Three marbles are selected simultane-
ously. Determine the probability that:
a all are red b only two are red c at least two are red.

7 Bag A contains 3 red and 2 white marbles.


Bag B contains 4 red and 3 white marbles.
RRR RR
One marble is randomly selected from A and its WW
RR
colour noted. If it is red, 2 reds are added to B. WWW
If it is white, 2 whites are added to B.
A marble is then selected from B. A B
What are the chances that the marble selected from B is white?

8 A man holds two tickets in a 100-ticket lottery in which there are two winning tickets.
If no replacement occurs, determine the probability that he will win:
a both prizes b neither prize c at least one prize.

I PROBABILITIES FROM VENN DIAGRAMS


(EXTENSION)

Example 13 Self Tutor


If the Venn diagram alongside illustrates the
T S
number of passengers on an aeroplane flight
who watched television (T) and slept (S), 46 4 8
determine the number of passengers:
12
U

a on the flight b who watched television


c who slept or watched television d who neither watched television
e who did not sleep nor slept

a Number on the flight b Number who watched television


= 46 + 4 + 8 + 12 = 70 = 46 + 4 = 50
c Number who slept or watched television
= 46 + 4 + 8 = 58
d Number who neither watched television nor slept = 12
e Number who did not sleep = 46 + 12 = 58

Page 254 of 525


100

100

100

100
50

50

50

50
0

0
25

75

95

25

75

95

25

75

95

25

75

95
5

IB_PD
cyan magenta yellow black

Y:\HAESE\IB_PD\IBPD_13\324ibpd13.cdr Wednesday, 5 July 2006 3:44:00 PM PETERDELL


PROBABILITY (Chapter 13) 325

EXERCISE 13I
1 The Venn diagram alongside illustrates the number of G
students in a particular class who study German (G)
5 S
and Spanish (S). Determine the number of students:
17
a in the class b who study both subjects 4
c who study at least one of the subjects 3
d who only study Spanish. U

2 In a survey people were asked if their holidays


for the last year had been within their own O A
country (O) or in another country (A). Use the
Venn diagram to determine the number of people: 37 9 15
a in the survey
b who had holidays in their own country 4
U
c who did not have holidays
d who only had holidays in another country.

Example 14 Self Tutor


In a class of 30 students, 19 play sport, 8 play music and 3 play sport and music,
in their leisure time. Display this information on a Venn diagram and hence deter-
mine the probability that a randomly selected class member plays:
a both sport and music b at least one of sport and music
c sport, but not music d exactly one of sport and music
e neither sport nor music f music if it is known that the student plays sport

S M Let S represent the event of ‘playing sport’,


a b c and M represent the event of ‘playing music’.
d Now a + b = 19 fas 19 play sportg
U
b+c = 8 fas 8 play musicg
S M
b=3 fas 3 play bothg
16 3 5
a + b + c + d = 30 fas there are 30 in the classg
U 6 ) b = 3, a = 16, c = 5, d = 6:
a P(plays both) b P(plays at least one of sport and music)
= 3
30 or 1
10 = 16+3+5
30

= 24
30 (or 45 )

c P(plays S, but not M) d P(plays exactly one)


16 16+5
= 30 = 30
8 7
= 15 = 10

e P(plays neither) f P(plays M if it is known plays S)


6 3
= 30 = 16+3
1 3
= 5 = 19

Page 255 of 525


100

100

100

100
50

50

50

50
0

0
25

75

95

25

75

95

25

75

95

25

75

95
5

IB_PD
cyan magenta yellow black

Y:\HAESE\IB_PD\IBPD_13\325ibpd13.cdr Wednesday, 5 July 2006 3:44:20 PM PETERDELL


326 PROBABILITY (Chapter 13)

3 In summer, in a class of 35 students, 18 swim, 14 play tennis and 8 do neither of these


sports. A student is randomly chosen from the class. Determine the probability that the
student:
a plays tennis b does not swim
c does at least one of the sports d does exactly one of the sports
e swims, but does not play tennis
4 On a hot day a group of 50 people at the beach were asked why they had come to the
beach. 27 had come to swim, 19 had come to surf, and 3 had come to do both. If one
person was chosen at random, determine the probability that he/she had come to:
a surf but not swim b neither surf nor swim
c swim, but not surf.
a+b
5 From the Venn diagram P(A) =
a+b+c+d A B
a Use the Venn diagram to find: a b c
i P(B) ii P(A and B) iii P(A or B)
d
iv P(A) + P(B) ¡ P(A and B)
b What is the connection between P(A or B) and P(A) + P(B) ¡ P(A and B)?

J CONDITIONAL PROBABILITY (EXTENSION)


THE ADDITION LAW
In question 5 of the previous exercise we showed that

for two events A and B, P(A [ B) = P(A) + P(B) ¡ P(A \ B).

This is known as the addition law of probability, and can be written as

P(either A or B) = P(A) + P(B) ¡ P(both A and B)

Example 15 Self Tutor

If P(A) = 0:6, P(A [ B) = 0:7


and P(A \ B) = 0:3, find P(B).
A B
U

Since P(A [ B) = P(A) + P(B) ¡ P(A \ B),


then 0:7 = 0:6 + P(B) ¡ 0:3
) P(B) = 0:4
Using a Venn diagram with the probabilities on it,
or a + 0:3 = 0:6 ) a = 0:3
a 0.3 b 0:3 + b + 0:3 = 0:7
) b = 0:1
A B
U ) P(B) = 0:3 + b = 0:4
Page 256 of 525
100

100

100

100
50

50

50

50
0

0
25

75

95

25

75

95

25

75

95

25

75

95
5

IB_PD
cyan magenta yellow black

Y:\HAESE\IB_PD\IBPD_13\326ibpd13.cdr Wednesday, 5 July 2006 3:44:58 PM PETERDELL


PROBABILITY (Chapter 13) 327

MUTUALLY EXCLUSIVE EVENTS


In a Venn diagram for
If A and B are mutually exclusive events then P(A \ B) = 0 mutually exclusive
events, the circles for the
and so the addition law becomes events do not overlap.
P(A [ B) = P(A) + P(B).

Mutually exclusive events


have no common outcomes. A B

Example 16 Self Tutor


A basket of fruit contains 8 apples (A) and 12 oranges (O).
a Are the events A and O mutually exclusive?
b Find i P(A) ii P(O) iii P(A \ O) iv P(A [ O).

a A piece of fruit cannot be an apple and an orange.


) A and O are mutually exclusive.

b i P(A) = 8
20 ii P(O) = 12
20 iii P(A \ O) iv P(A [ O)
2 3 =0 = 20
= 5 = 5 20
=1

CONDITIONAL PROBABILITY
If we have two events A and B, then

A j B is used to represent that ‘A occurs knowing that B has occurred’.

Example 17 Self Tutor


In a class of 25 students, 15 like chocolate milk (M) and 17 like iced coffee (C).
Two students like neither and 9 students like both. One student is randomly selected
from the class. What is the probability that the student:
a likes chocolate milk
b likes chocolate milk given that he/she likes iced coffee?

The Venn diagram of the situation is shown.


M C
a 15 of the 25 students like chocolate milk.
6 9 8
) P(chocolate milk) = 15 25

U 2 b Of the 17 who like iced coffee, 9 like chocolate milk.


) P(chocolate milk j iced coffee) = 9
17

Page 257 of 525


100

100

100

100
50

50

50

50
0

0
25

75

95

25

75

95

25

75

95

25

75

95
5

IB_PD
cyan magenta yellow black

Y:\HAESE\IB_PD\IBPD_13\327ibpd13.cdr Wednesday, 5 July 2006 3:45:23 PM PETERDELL


328 PROBABILITY (Chapter 13)

If A and B are events then Proof: P(A j B)


A
a b
P(A \ B) = fVenn diagramg
P(A j B) = . b+c
P(B)
b b ¥ (a + b + c + d)
=
(b + c) ¥ (a + b + c + d)
B c
P(A \ B)
U d =
P(B)

It follows that P(A \ B) = P(A j B) P(B) or P(A \ B) = P(B j A) P(A)

Example 18 Self Tutor


In a library group of 50 readers, 34 like science fiction, 22 like detective stories
and 12 dislike both.
If a reader is randomly selected, find the probability that the reader:
a likes science fiction and detective stories
b likes at least one of science fiction and detective stories
c likes science fiction given that he/she likes detective stories
d dislikes detective stories given that he/she likes science fiction.

S D
S represents readers who like science fiction
a b c D represents readers who like detective stories
U 12 We are given that a + b = 34
b + c = 22
S D a + b + c = 38
16 18 4
) c = 38 ¡ 34 and so b = 18
U 12
=4 and a = 16
a P(likes both) b P(likes at least one) c P(S j D) d P(D0 j S)
18 38 18 16
= 50 = 50 = 22 = 34
9 19 9 8
= 25 = 25 = 11 = 17

EXERCISE 13J
1 50 students went on a ‘thrill seekers’ holiday. 40 went white-water rafting, 21 went
paragliding and each student did at least one of these activities.
a From a Venn diagram find how many students did both activities.
b If a student from this group is randomly selected, find the probability that he/she:
i went white-water rafting but not paragliding
ii went paragliding given that he/she went white-water rafting.

2 In a class of 25 students, 19 have fair hair, 15 have blue eyes, and 22 have fair hair, blue
eyes or both. A child is selected at random. Determine the probability that the child has:
a fair hair and blue eyes b neither fair hair nor blue eyes
c fair hair but not blue eyes d blue eyes given that the child has fair hair.
Page 258 of 525
100

100

100

100
50

50

50

50
0

0
25

75

95

25

75

95

25

75

95

25

75

95
5

IB_PD
cyan magenta yellow black

Y:\HAESE\IB_PD\IBPD_13\328ibpd13.cdr Wednesday, 5 July 2006 3:45:42 PM PETERDELL


PROBABILITY (Chapter 13) 329

3 28 students go tramping. 23 get sunburnt, 8 get blis-


ters and 5 get both sunburnt and blisters. Determine
the probability that a randomly selected student:
a did not get blisters
b either got blisters or sunburnt
c neither got blisters nor sunburnt
d got blisters, given that the student was sunburnt
e was sunburnt, given that the student did not get
blisters.
4 An examination in French has two parts; aural and written. When 30 students sit for
the examination, 25 pass aural, 26 pass written and 3 fail both parts. Determine the
probability that a student who:
a passed aural also passed written b passed aural, failed written.

5 In a certain town there are 3 supermarkets, P, Q and R. 60% of the population shop at P,
36% shop at Q, 34% shop at R, 18% shop at P and Q, 15% shop at P and R, 4% shop
at Q and R and 2% shop at all 3 supermarkets. A person is selected at random.
Determine the probability that the person shops at:
a none of the supermarkets b at least one of the supermarkets
c exactly one of the supermarkets d either P or Q
e P, given that the person shops at at least one supermarket
f R, given that the person shops at either P or Q or both.

Example 19 Self Tutor


Bin A contains 2 red and 4 white tickets. Bin B contains 3 red and 3 white. A die
with four faces marked A and two faces marked B is rolled and used to select bin
A or B. A ticket is then selected from this bin. Determine the probability that:
a the ticket is red b the ticket was chosen from B given it is red.

a P(R)
bin ticket = 46 £ 26 + 2
£ 3
fthe X pathsg
6 6
Wy_ R X 1 = 14
36
A 7
Ry_ = 18
Ry_ W
b P(B j R)
Wy_
Ey_ R X 2 P(B \ R)
B =
P(R)
Ey_ W
2 3
6 £ 6 path 2
= 7
18 paths 1 and 2
3
= 7

Page 259 of 525


100

100

100

100
50

50

50

50
0

0
25

75

95

25

75

95

25

75

95

25

75

95
5

IB_PD
cyan magenta yellow black

Y:\HAESE\IB_PD\IBPD_13\329ibpd13.cdr Wednesday, 5 July 2006 3:45:58 PM PETERDELL


330 PROBABILITY (Chapter 13)

6 Marius has 2 bags of peaches. Bag A has 4 ripe and 2 unripe peaches, and bag B has
5 ripe and 1 unripe peaches. Ingrid selects a bag by tossing a coin, and takes a peach
from that bag.
a Determine the probability that the peach is ripe.
b Given that the peach is ripe, what is the probability it came from B?

7 When Sophia goes to the local shopping centre on Monday to Thursday the probability
that she finds a carpark is 95%. When she goes on Friday or Saturday the probability of
finding a carpark is 70%. Assuming that she is equally likely to shop on any day from
Monday to Saturday, determine the probability that on any trip:
a she finds a carpark b it is Saturday, given that she finds a carpark.

8 On a given day, Claude’s car has an 80% chance of starting first time and André’s car
has a 70% chance of the same. Given that at least one of the cars has started first time,
what is the chance that André’s car started first time?

REVIEW SET 13A


1 Donna keeps records of the number of clients she number of days
interviews over a consecutive period of days. 12
a For how many days did the survey last? 9
b Estimate Donna’s chances of interviewing: 6
i no clients on a day
3
ii four or more clients on a day
iii less than three clients on a day. 0 1 2 3 4 5 6 7
number of clients
2 Illustrate on a 2-dimensional grid the possible outcomes when a coin and a pentagonal
spinner with sides labelled A, B, C, D and E are tossed and spun.
3 What is meant by saying that two events are “independent”?
4 Use a tree diagram to illustrate the sample space for the possible four-child families and
hence determine the probability that a randomly chosen four-child family:
a is all boys b has exactly two boys c has more girls than boys.
5 In a shooting competition, Louise has 80% chance of hitting her target and Kayo has
90% chance of hitting her target. If they both have a single shot, determine the
probability that:
a both hit their targets b neither hits her target
c at least one hits her target d only Kayo hits her target
6 Two fair six-sided dice are rolled simultaneously. Determine the probability that the
result is a double.
7 A bag contains 4 green and 3 red marbles. Two marbles are randomly selected from the
bag, the first being replaced before the drawing of the second. Determine the probability
that:
a both are green b they are different in colour.

Page 260 of 525


100

100

100

100
50

50

50

50
0

0
25

75

95

25

75

95

25

75

95

25

75

95
5

IB_PD
cyan magenta yellow black

Y:\HAESE\IB_PD\IBPD_13\330ibpd13.cdr Wednesday, 5 July 2006 3:46:13 PM PETERDELL


PROBABILITY (Chapter 13) 331

8 A circle is divided into 5 sectors with congruent angles at the centre and a spinner is
constructed. The sectors are numbered 1, 2, 3, 4, and 5. A coin is tossed and the spinner
is spun.
a Use a 2-dimensional grid to show the sample space.
b What is the chance of getting: i a head and a 5 ii a head or a 5?

9 Bag X contains three white and two red marbles. Bag Y contains one white and three
red marbles. A bag is randomly chosen and two marbles are drawn from it. Illustrate
the given information on a tree diagram and hence determine the probability of drawing
two marbles of the same colour.

10 At a local girls’ school, 65% of the students play netball, 60% play tennis and 20%
play neither sport. Display this information on a Venn diagram and hence determine the
likelihood that a randomly chosen student plays:
a netball b netball but not tennis
c at least one of these two sports d exactly one of these two sports
e tennis, given that she plays netball

REVIEW SET 13B


1 Pierre conducted a survey to determine the ages of peo- Age Frequency
ple walking through a shopping mall. The results are
0 - 19 22
shown in the table alongside. Find, correct to 3 deci-
20 - 39 43
mal places, the estimated probability that the next person
40 - 59 39
Pierre meets in the shopping mall is:
60+ 14
a between 20 and 39 years of age
b less than 40 years of age c at least 20 years of age.

2 a List, in set notation, the sample space of possible results when a tetrahedral die
with four faces labelled A, B, C and D is rolled and a 20-cent coin are tossed
simultaneously.
b Use a tree diagram to illustrate the sample space for the following:
Bags A, B and C contain green or yellow tickets. A bag is selected and then a
ticket taken from it.

3 Use a tree diagram to illustrate the following sample space: Martina and Justine play
tennis. The first to win three sets wins the match.

4 When a box of drawing pins dropped onto the floor it was observed
that 49 landed on their backs and 32 landed on their sides. Find,
correct to 2 decimal places, the estimated probability of a drawing pin back side
landing:
a on its back b on its side.

Page 261 of 525


100

100

100

100
50

50

50

50
0

0
25

75

95

25

75

95

25

75

95

25

75

95
5

IB_PD
cyan magenta yellow black

Y:\HAESE\IB_PD\IBPD_13\331ibpd13.cdr Wednesday, 5 July 2006 3:46:26 PM PETERDELL


332 PROBABILITY (Chapter 13)

5 The letters A, B, C, D, ... N are put in a hat.


a Determine the probability of drawing a vowel (A, E, I, O or U) if one of the
letters is chosen at random.
b If two letters are drawn without replacement,
1st draw 2nd draw
copy and complete the following tree diagram
vowel
including all probabilities where appropriate: vowel
non-vowel
c Use your tree diagram to determine the
vowel
probability of drawing: non-vowel
i a vowel and a non-vowel non-vowel
ii at least one vowel.

6 A farmer fences his rectangular property, as shown alongside,


into 9 rectangular paddocks:
If a paddock is selected at random, what is the probability that
a it has no fences on the boundary of the property
b it has one fence on the boundary of the property
c it has two fences on the boundary of the property?

7 If P(A) = 12 ,
7
find P(A0 ) . (Recall: A0 is the complement of A.)

8 Bag X contains 3 black and 2 red marbles. Bag Y contains 4 black and 1 red marble.
A bag is selected at random and then two marbles are selected without replacement.
Determine the probability that:
a both marbles are red b two black marbles are picked from Bag Y.

9 Two dice are rolled simultaneously. Illustrate this information on a 2-dimensional grid.
Determine the probability of getting:
a a double 5 b at least one 4 c a sum greater than 9
d a sum of 7 or 11

10 A class consists of 25 students, 15 have blue eyes, 9 have fair hair and 3 have both
blue eyes and fair hair. Represent this information on a Venn diagram and hence find
the probability that a randomly selected student from the class:
a has neither blue eyes nor fair hair b has blue eyes, but not fair hair
c has fair hair given that he or she has blue eyes
d does not have fair hair given that he or she does not have blue eyes.

Page 262 of 525


100

100

100

100
50

50

50

50
0

0
25

75

95

25

75

95

25

75

95

25

75

95
5

IB_PD
cyan magenta yellow black

Y:\HAESE\IB_PD\IBPD_13\332ibpd13.cdr Wednesday, 5 July 2006 3:46:35 PM PETERDELL

You might also like